Section 11 Pulmonology
SECTION 11 Pulmonology
912
CHAPTER 45
913
Hemoptysis
914 Case: A 29-year-old woman with a diastolic murmur A 29-year-old woman who emigrated from Mexico at 10 years of age presen s to the emergency department with sudden-onset sever
abdominaltpain. Three daystprior, the patient)experienced transiente hemiparesthesias and dysar hria. Cross-sectional imaging of the abdomen (Figure 45-1) and chest (Figure 45-2 are shown.
FIGURE 45-1
FIGURE 45-2
After the procedure, she develops acute-onset dyspnea, hypoxemia, . one-half cup over the course of an hour. The patient is taken for emergency right renal artery embolectomy and hemoptysis. The sputum is bright red in color with a volume of Cardiac auscultation reveals a regular rhythm with a pronounced S1, an extra sound just after S2 best heard with the diaphragm of the presystolic accentuation best heard with the bell of the stethoscope stethoscope over the apex, and a rumbling diastolic murmur with over the apex. Phonocardiograms of the heart sounds are shown in Figure 45-3. For audio of the heart sounds in this case, see the associated reference.1
FIGURE 45-3 Phonocardiogram from a contemporary electronic stethoscope (top). Phonocardiogram from a mid-20th century antique phonocardiograph
915
(bottom). (Reprinted with permission from Oehler AC, Sullivan PD, Mansoor AM. Mitral stenosis. BMJ Case Rep. 2017;2017.)
What is the most likely cause of hemoptysis in this patient?
What is hemoptysis?
What conditions can mimic hemoptysis and should always be considered in a patient who expectorates blood? What historical features can help distinguish hemoptysis from hematemesis?
Which arteries supply blood to the lungs?
Hemoptysis is the expectoration of blood produced within the lower respiratory tract, anywhere from the trachea to the alveoli.2 Hematemesis, expectorated blood from a source originating in the upper Gastrointestinal tract, and bleeding from the upper respiratory tract are often confused for hemoptysis. Epistaxis with drainage down the throat into the lungs is common, underscoring the importance of nasal examination in patients with suspected hemoptysis. The following features are suggestive of hematemesis: nausea and vomiting; preexisting gastric or hepatic disease; and brown, black, or coffee ground appearance of the expectorated material. The following are suggestive of hemoptysis: absence of nausea and vomiting, preexisting lung disease associated with hemoptysis (eg, bronchiectasis), and frothy, bright red, liquid, or clotted appearance of the expectorated material.3 The pulmonary arteries supply 99% of blood to the lungs, while the bronchial arteries supply the remaining 1%. The pulmonary arteries are involved in gas exchange, while the bronchial arteries supply circulation to the airways, lung parenchyma, and pulmonary arteries (Figure 45-4). Capillary anastomoses between the 2 systems exist.4
FIGURE 45-4 Blood supply of the lungs. The bronchial circulation contributes to the pulmonary microvasculature by supplying blood to the bronchovascular bundles, pulmonary interstitium, and the vasa vasorum of the pulmonary arteries and veins. (From Saremi F. Perfusion Imaging in Clinical Practice: A Multimodality Approach to Tissue Perfusion Analysis. Philadelphia, PA: Wolters Kluwer; 2015.)
Which arterial system is Both the pulmonary arteries and the bronchial arteries may be involved in hemopytsis, but the higher-pressure involved in hemoptysis? bronchial arteries are the most common source of massive hemoptysis (around 90% of cases) compared with the lower-pressure pulmonary arteries (around 5% of cases); pulmonary and bronchial capillaries and veins cause the remaining cases.4 How is the severity of The severity of hemoptysis is usually graded as massive or submassive. No universal criteria have been hemoptysis defined? established, but volume thresholds between 100 and 1000 mL over a 24-hour period have been suggested to define massive hemoptysis. A reasonable threshold to use is 300 mL/d. Quantification of hemoptysis is not always possible or practical, underscoring the importance of clinical judgment.4 Why is it important to Massive hemoptysis is a life-threatening condition with a significant mortality rate when it is not promptly determine the quantity managed. The first step is to maintain a safe airway and then identify the source of bleeding with chest of hemoptysis? imaging and/or bronchoscopy. The patient should be positioned such that the side of bleeding is gravity- dependent (ie, bleeding side down) until definitive management occurs (usually arterial endovascular embolization or surgery).4 What is the role of Chest radiography is important in the initial workup of hemoptysis because it is widely available, inexpensive, conventional chest and prompt. The sensitivity of conventional radiography for localizing the source of bleeding is approximately radiography in patients 50%.4 with hemoptysis? What is the role of In patients with hemoptysis, CT imaging with intravenous contrast is more sensitive than conventional computed tomography radiography for localizing the source of bleeding. It also has the advantage of providing a more detailed (CT) imaging in patients evaluation of lung anatomy and may be able to diagnose the underlying etiology (eg, bronchiectasis).4 with hemoptysis? What is the role of In patients with hemoptysis, bronchoscopy is complementary to chest imaging in detecting the site and bronchoscopy in patients underlying cause of bleeding. Advantages of bronchoscopy include the ability to localize endobronchial with hemoptysis? lesions, although massive bleeding may obscure a bleeding site. Sampling for culture and histopathology are additional advantages. Sometimes bleeding can be controlled with bronchoscopic techniques (eg, cold saline, balloon inflation, or laser coagulation).4 Which 2 systems are Most cases of hemoptysis involve the cardiovascular or pulmonary systems. involved in most cases of hemoptysis?
916
917
Cardiovascular Causes of Hemoptysis
What are the cardiovascular causes of hemoptysis?
Acute pulmonary arterial hypertension. Pulmonary embolism (PE) and infarction (Figure 45-5).
FIGURE 45-5 Contrast-enhanced CT showing bilateral filling defects in the
Elevated left atrial pressure leading to rupture of the alveolar capillaries. This group of inflammatory conditions often presents with protean manifestations, reflecting the wide range of potential organ system involvement. Associated with a murmur on auscultation of the heart. A 32-year-old woman with cutaneous telangiectasias and episodes of hemoptysis and epistaxis.
How common is hemoptysis in patients with pulmonary embolism?
What are the characteristics of hemoptysis associated with congestive heart failure?
pulmonary arteries (arrows) in a patient with pulmonary embolism. (From Brant WE, Helms CA. Fundamentals of Diagnostic Radiology. 4th ed. Philadelphia, PA: Lippincott Williams & Wilkins; 2012.)
Heart failure. Vasculitis.
Valvular disease. Arteriovenous malformation (AVM).
Hemoptysis is more likely in patients with PE who have an associated pulmonary infarction, but occurs in a minority of patients overall. More common manifestations of PE include dyspnea, pleuritic chest pain, tachypnea, tachycardia, and syncope.5 Hemoptysis caused by congestive heart failure is nonmassive and often described as “pink, frothy” sputum.
Which vasculitides are associated with hemoptysis?
Which valvular lesions are associated with hemoptysis? What underlying genetic condition commonly causes pulmonary arteriovenous malformations?
Hemoptysis is generally associated with the antineutrophil cytoplasmic antibody–associated small vessel systemic vasculitides, including granulomatosis with polyangiitis (GPA, or Wegener’s granulomatosis), microscopic polyangiitis, and eosinophilic granulomatosis with polyangiitis (EGPA, or Churg-Strauss syndrome). Pulmonary capillaritis can also be associated with antiglomerular basement membrane disease (ie, Goodpasture syndrome) and underlying rheumatologic diseases (eg, systemic lupus erythematosus). Any valvular lesion that results in left-sided heart failure can lead to hemoptysis via pulmonary venous hypertension. However, 3 lesions can cause hemoptysis via other mechanisms and deserve special attention: tricuspid valve endocarditis, which can result in septic pulmonary emboli; mitral stenosis, which can result in sudden elevation of left atrial pressure; and mitral regurgitation with flail posterior leaflet, which can result in asymmetric right upper lobe pulmonary edema and a focal increase in pulmonary capillary pressure. Hereditary hemorrhagic telangiectasia (HHT, or Osler-Weber-Rendu syndrome) is an autosomal dominant condition characterized by the development of vascular malformations in various organs of the body. Patients most often present with cutaneous telangiectasias (see Figure 40-2) and frequent epistaxis. Some patients develop pulmonary AVMs that are at risk for spontaneous rupture, which can result in massive hemoptysis or hemothorax.
918
919
Pulmonary Causes of Hemoptysis
The pulmonary causes of hemoptysis can be separated into which 2 anatomic subcategories?
Pulmonary causes of hemoptysis can involve the airways or parenchyma.
What are the causes of hemoptysis related to the airways?
Most commonly caused by a viral or bacterial infection. Dilation and thickening of the airways on chest imaging (see Figure20-3). A 66-year-old man with a history of chronic obstructive pulmonary disease (COPD) and active cigarette use presents with subacute weight loss, night sweats, and hemoptysis. A 24-year-old man emerges from a bar fight with missing teeth and a new cough. A patient develops hemoptysis 2 hours after bronchoscopy with endobronchial biopsy. Communication between blood vessel and airway.
Acute bronchitis. Bronchiectasis. Malignancy (eg, bronchogenic carcinoma).
Foreign body aspiration. Iatrogenic.
Bronchovascular fistula.
How often is bronchitis the cause of Acute bronchitis accounts for up to one-half of cases of hemoptysis in the industrialized hemoptysis? world. In addition to infection, inhalation of various gases, fumes, or dusts (ie, inhalation injury) can cause acute bronchitis.6,7 What are the mechanisms of hemoptysis Hemoptysis results from denudation and proliferative neovascularization of the airway in the setting of bronchiectasis? mucosa that occurs in patients with bronchiectasis. Airway dilation, which brings the bronchial arteries closer to the mucosal surface, is a contributing factor.2 How often is hemoptysis part of the initial Only around 10% of patients with primary lung cancer present with hemoptysis. It is more presentation in patients with primary lung likely to occur in cancers arising centrally, such as squamous cell carcinoma and small cell cancer? lung cancer. Massive hemoptysis can occur in these patients when there is erosion into the hilar blood vessels. Endobronchial tumors, such as carcinoid, can also cause hemoptysis.2 How can the presence of an aspirated Depending on the foreign body, conventional chest radiography can confirm aspiration in foreign body be confirmed? most cases. CT is more sensitive for small objects. Bronchoscopy can be both diagnostic and therapeutic.8 What are the iatrogenic causes of Hemoptysis can be caused by bronchoscopy with endobronchial or transbronchial biopsy, hemoptysis? vascular injury related to catheters, percutaneous lung biopsy, airway stents, and bone marrow transplantation (BMT). What is the nature and timing of Some patients present with sudden-onset massive hemoptysis within 100 days of BMT as a hemoptysis in patients who have result of diffuse alveolar hemorrhage. The pathogenesis of this complication has not been undergone bone marrow transplantation? completely elucidated.2 What are the characteristics of hemoptysis Massive hemoptysis is typical of bronchovascular fistulae, sometimes preceded by a sentinel related to a bronchovascular fistula? bleed. Bronchovascular fistulae usually occur as a result of invasive infection (eg, aspergillosis) or pulmonary procedures (eg, lung resection, lung transplantation).9
920
What are the causes of hemoptysis related to the pulmonary parenchyma?
Infection of the alveoli. A 56-year-old man with a history of heavy alcohol abuse presents with low-grade fever and malaise, and chest imaging reveals parenchymal consolidation with an air-fluid level (see Figure31- 4). The leading cause of hemoptysis worldwide. A 33-year-old woman with a history of heart transplantation for familial dilated cardiomyopathy presents with fever, weight loss, and hemoptysis and is found to have a crescentic cavitation on chest imaging. A 63-year-old man presents with fever, cough, and hemoptysis 8 weeks after eating freshwater crabs.
Hemoptysis most commonly occurs in association with pneumonia caused by which organisms?
What organisms are commonly associated with lung abscess?
How common is hemoptysis in patients with active pulmonary tuberculosis?
Which populations are at highest risk of acquiring fungal infection of the lungs?
Where is Paragonimus westermani endemic?
Pneumonia. Lung abscess.
Pulmonary tuberculosis (TB).6 Pulmonary aspergillosis.
Pulmonary paragonimiasis.
Organisms that cause necrotizing pneumonia are more likely to result in hemoptysis, including Staphylococcus aureus and gram-negative rods (eg, Klebsiella pneumoniae, Pseudomonas aeruginosa). Distortion of lung architecture that might occur as a result of severe pneumonia can predispose patients to hemoptysis during subsequent infections.2 Lung abscesses are usually polymicrobial. Anaerobic organisms of the oral cavity are commonly involved, including gram-negatives (eg, Bacteroides species) and gram-positives (eg, Peptostreptococcus species). Aerobic organisms (eg, Staphylococcus aureus) are less commonly involved.10 Hemoptysis develops in around one-quarter of patients with active pulmonary TB. Most patients present with blood-streaked sputum. However, massive hemoptysis can occur when cavitations erode into surrounding vascular structures such as the bronchial arteries.2 Immunocompromised patients and those with chronically diseased lungs (eg, COPD) are at highest risk for acquiring pulmonary fungal infections. As with pulmonary TB, massive hempoptysis can occur when cavitations erode into surrounding vascular structures.6 Paragonimus westermani is a lung fluke endemic to Southeast Asia and China. It typically presents with fever, cough, and hemoptysis, and is frequently mistaken for pulmonary TB. Cases in the United States have been reported in patients who have ingested crayfish or small crabs.2
921
Other Causes of Hemoptysis
What are the other causes of hemoptysis?
“Hemoptysis in a woman is removed by eruption of the menses.”—Hippocrates. More common in children, this condition should be considered when there are recurrent episodes of diffuse alveolar hemorrhage without an identifiable cause.
What is the pathogenesis of catamenial hemoptysis?
What is the long-term effect of idiopathic pulmonary hemosiderosis on the lungs?
Catamenial hemoptysis.11
Idiopathic pulmonary hemosiderosis.
Catamenial hemoptysis results from the presence of ectopic endometrial tissue within the lower respiratory tract, including the parenchyma and airways, possibly as a result of entry through small defects in the diaphragm. Hormonal suppression of endometrial proliferation may be effective in some cases. Surgery can be offered in refractory cases, but there is risk of recurrence.12 Idiopathic pulmonary hemosiderosis is a rare condition of unknown etiology, characterized by recurrent episodes of diffuse alveolar hemorrhage. Bleeding can be severe and life-threatening. The accumulation of iron in the alveoli over time can lead to pulmonary fibrosis. Glucocorticoids have been shown to reduce the development of pulmonary fibrosis, as well as overall morbidity and mortality. There may also be a role for other immunosuppressive medications.13
922 Case Summary A 29-year-old woman from Mexico develops acute-onset
hemoptysis after undergoing a thrombectomy procedure and is found to have an abnormal cardiac examination, including an extra heart
sound and a rumbling diastolic murmur.1 What is the most likely cause of hemoptysis in this patient? Mitral valve stenosis.
923
Bonus Questions
What is the significance of the Embolic phenomena occur in patients with mitral stenosis and can be part of the initial presentation. Emboli are usually, but not always, related to embolic events in this case? concomitant atrial fibrillation. What is the most likely source of Extra heart sounds that occur near S2 include split S2, S3 gallop, opening snap, pericardial knock, and tumor plop. The extra sound in this case ( 1 the extra heart sound in this case? Figure 45-3) is most likely the opening snap of mitral stenosis, based on location, pitch, the associated diastolic murmur, and clinical history. Forsee . audio of the heart sounds in this case, see the associated reference 1,14 What 4 main auscultatory findings The main auscultatory findings of mitral stenosis include the following: (1) pronounced S1, (2) early diastolic opening snap, (3) rumbling diastolic are found in patients with murmur at the apex best heard with the bell of the stethoscope, and (4) presystolic accentuation of the murmur. An irregularly irregular heart significant mitral stenosis? rhythm will be present in patients with atrial fibrillation.1 What cardiac event causes the At the end of diastole, left atrial contraction against a stenotic mitral valve increases turbulent blood flow, augmenting the intensity of the murmur of mitral stenosis? presystolic accentuation of associated murmur. This finding is absent in patients with atrial fibrillation because there is no coordinated atrial contraction. What long-term sequela of mitral CT imaging of the chest in this case (see Figure 45-2) demonstrates that the pulmonary trunk (star) is significantly larger than the adjacent
stenosis is present on CT imaging in ascending aorta (arrow), a clue to the presence of pulmonary hypertension. The ratio of the diameter of the main pulmonary artery to that of the this case? ascending aorta is normally <1 in adults. 15 What are the causes hemoptysis in The hemoptysis of mitral stenosis occurs via 2 primary mechanisms: (1) pulmonary edema related to pulmonary venousihypertension, resulting in patients with mitral stenosis? pink, frothy sputum and (2) rupture of the thin-walled bronchial veins (ie, pulmonary apoplexy) from sudden increases n left atrial pressure, resulting in massive but typically self-limited hemoptysis. 16,17 What likely triggered hemoptysis in Intthis case, it is likely that hemodynamic changes related to renal artery embolism or the embolectomy procedure resulted in a suddeniincrease in What is the most common cause of Rheumatic heart disease is the most common cause of mitral stenosis worldwide.16 this case? left atrial pressure. Unlike aortic stenosis, mitral stenosis is associated with minimal functional reserve, and decompensation occurs eas ly in the se ting of tachycardia or high flow. 16 mitral stenosis worldwide? What is the prognosis of severe Without treatment, the prognosis of,severe mitral stenosis is poor. Morbidity and mortality are significantly improved with treatment, including mitral stenosis? percutaneous balloon valvuloplasty surgical valvotomy, or mitral valve replacement. 16
924 Key Points
Hemoptysis is the expectoration of blood produced in the lower Hematemesis, bleeding from the upper airway, and nasal bleeding The severity of hemoptysis is graded as massive or submassive; management, includingimaintaining the airway and placing the Conventional radiography, CT imaging, and bronchoscopy play The causes of hemoptysis can be separated into the following Pulmonary causes of hemoptysis can involve the airways or respiratory tract. can be confused with hemoptysis and should be ruled out. 300 mL/d is a reasonable threshold to distinguish the two. Massive hemoptysis is l fe-threatening and requires prompt bleeding lung in a dependent position. complementary roles in the evaluation of hemoptysis. categories: cardiovascular, pulmonary, and other. parenchyma. The most common causes of hemoptysis include acute bronchitis,
pneumonia, lung abscess, lung cancer, tuberculosis, heart failure, and bronchiectasis.
925
References 1. Oehler AC, Sullivan PD, Mansoor AM. Mitral stenosis. BMJ Case Rep. 2017;2017. 2. Longo DL, Fauci AS, Kasper DL, Hauser SL, Jameson JL, Loscalzo J, eds. Harrison’s Principles of Internal Medicine. 18th ed. New York, NY: McGraw-Hill; 2012.
-
Bidwell JL, Pachner RW. Hemoptysis: diagnosis and management. Am Fam Physician. 2005;72(7):1253-1260.
-
Larici AR, Franchi P, Occhipinti M, et al. Diagnosis and management of hemoptysis. Diagn Interv Radiol. 2014;20(4):299-309.
-
Kostadima E, Zakynthinos E. Pulmonary embolism: pathophysiology, diagnosis, treatment. Hellenic J Cardiol. 2007;48(2):94-107.
-
Kapur S, Louie BE. Hemoptysis and thoracic fungal infections. Surg Clin North Am. 2010;90(5):985-1001.
-
Reisz G,iStevens D, Boutwell C, Nair V. The causes of hemoptysis revisited.)A review of the et ologies of hemoptysis between 1986 and 1995. Mo Med. 1997;94(10 :633-635.
-
Qureshi A, Behzadi A. Foreign-body aspiration in an adult. Can J Surg. 2008;51(3):E69- E70.
-
Sellke FW, del Nido PJ, Swanson SJ, eds. Sabiston & Spencer Surgery of the Chest. 9th ed. Philadelphia, PA: Elsevier, Inc.; 2015.
-
Kuhajda I, Zarogoulidis K, Tsirgogianni K, et al. Lung.abscess-etiology, diagnostic 11. Eastman TJ. Periodical hemoptysis. Boston Med Surg J. 1910(162):320-322. and treatment options. Ann Transl Med. 2015;3(13):183
-
Augoulea A, Lambrinoudaki I, Christodoulakos G. Thoracic endometriosis syndrome. Respiration. 2008;75(1):113-119.
-
Ioachimescu OC, Sieber S, Kotch A. Idiopathic pulmonary haemosiderosis revisited. Eur Respir J. 2004;24(1):162-170.
-
Marriott HJL. Bedside Cardiac Diagnosis. Philadelphia, Pennsylvania: Lippincott Company; 1993.
-
Pena E,.Dennie C, Veinot J, Muniz:SH. Pulmonary hypertension: how the radiologist 16. Chandrashekhar Y, Westaby S, Narula J. Mitral stenosis. Lancet. 2009;374(9697):1271- can help Radiographics. 2012;32(1) 9-32. 1283.
-
Scarlat A, Bodner G, Liron M. Massive haemoptysis as the presenting symptom in mitral stenosis. Thorax. 1986;41(5):413-414.
926
CHAPTER 46
927
Hypoxemia
928 Case: A 51-year-old man with positional dyspnea A 51-year-old man with a history of chronic hepatitis C infection
complicated by cirrhosis presents to the emergency department with shortness of breath. He has been followed by a hepatologist for management of hepatic encephalopathy, ascites, and esophageal varices which have been stable for several years. The patient has a
history,of shortness of breath that typically resolves following large volume paracentesis, which he has occasionally required. However, over the past few months the patient has noticed progressive dyspnea
despite control of the ascites. Breathing seems to improve when he lies peHeart rate is 100 beats per minute, and respiratory rate is 24 breaths on his back. r minute. Hemoglobin oxygen saturation by pulse oximetry (Sp o 2 ) is 85% on room air with the patient in the upright position; it improves to 96% in the supine position. There are multiple spider angiomas one 2 the anterior chest. Jugular venous pressure is 7 cm H O. The lungs ar clear to auscultation. An image of the patient’s hand is shown in prArterial blood gas on room air shows that pH is 7.48, partial ssure of with agitated saline contrast shows the presence of microbubbles in same study is captured 8 cardiac cycles after initial opacification of the Figure 46-1. essure of carbon dioxide (Pa co 2 ) is 32 mm Hg, and partial pre oxygen is 56 mm Hg. An image from a transthoracic echocardiogram the right-sided cardiac chambers (Figure 46-2A). An image from the right-sided cardiac chambers (Figure 46-2B).
FIGURE 46-1
FIGURE 46-2 (Courtesy of Steven E. Mansoor, MD, PhD.)
929
What is the most likely cause of hypoxemia in this patient?
What is hypoxemia?
Which noninvasive bedside instrument can be used to estimate the partial pressure of oxygen in arterial blood by spectrophotometrically measuring the fraction of oxygenated hemoglobin in peripheral arterial blood? How accurate is pulse oximetry?
Under what conditions is pulse oximetry unreliable?
Which invasive test can directly measure the partial pressure of oxygen in arterial blood? How does the partial pressure of oxygen in arterial blood relate to the true oxygen saturation of hemoglobin in arterial blood?
What physiologic conditions shift the oxyhemoglobin dissociation curve to the right, and what conditions shift the curve to the left? What formula describes the oxygen content of arterial blood (Cao2)?
Using the formula for the oxygen content of arterial blood as reference, what are the 2 main determinants of the oxygen content of arterial blood? Which formula describes oxygen delivery (Do2) to the tissues? Using the formula for oxygen delivery as reference, what are the 3 main determinants of
Hypoxemia refers to a physiologic state in which the partial pressure of oxygen in arterial blood (Pao2) is low. Normal Pao2 at room air is 80 to 100 mm Hg.1,2 Pulse oximetry measures the fraction of oxygenated hemoglobin in peripheral arterial blood (expressed as % saturation, Spo2), which can be used to provide an estimate of Pao2.
Pulse oximetry is accurate to within ±3% when true oxygen saturation of hemoglobin in arterial blood (Sao2) is above 70%.3 Pulse oximetry is unreliable in patients with carbon monoxide poisoning and methemoglobinemia. In the setting of methemoglobinemia, the reading rarely falls below 85%, which can be a clue to the diagnosis. Measurement of pulse oximetry requires a steady and fairly regular pulse, and therefore may be unreliable in the setting of poor circulation (eg, Raynaud’s phenomenon) and cardiac dysrhythmias (eg, atrial fibrillation). Other conditions may compromise measurements by obstructing the pulse oximeter (eg, the presence of fingernail polish). The use of an oximetry probe on the earlobe may be helpful in some circumstances.3 Pao2 can be directly measured with an arterial blood gas test. Sao2 can be calculated based on the Pao2 value and other factors (eg, blood pH). Under normal circumstances, Sao2 is dependent on Pao2, and a predictable relationship has been established (oxygen-hemoglobin [or oxyhemoglobin] dissociation curve). Important numbers to recognize are Pao2 of 27 mm Hg and Pao2 of 60 mm Hg, which correspond to hemoglobin saturations of 50% and 90%, respectively (Figure 46-3).
FIGURE 46-3 Oxyhemoglobin dissociation curve showing the normal relationship between the Po2 in blood and the O2 saturation of hemoglobin. The P50 is the Po2 that corresponds to 50% saturation of hemoglobin with O . 2 (From Marino PL. Marino’s The ICU Book. 4th ed. Philadelphia, PA: Wolters Kluwer Health/Lippincott Williams & Wilkins; 2014.)
Conditions that shift the oxyhemoglobin dissociation curve to the right include Increased temperature, acidemia, Increased Paco2, and Increased 2,3-bisphosphoglycerate (2,3-BPG). Conditions that shift the curve to the left include Decreased temperature, alkalemia, Decreased Paco2, and Decreased 2,3-BPG (see Figure 46-3).4 Cao2 =Sao2 ×[Hb] ×1.34 +(0.003 ×Pao2) In the above formula, Cao2 is expressed in mL O2 /dL blood, Sao2 is expressed as a fraction, [Hb] is the hemoglobin concentration in blood (g/dL), 1.34 is the oxygen-binding capacity of hemoglobin (mL O2/g Hb), 0.003 is the solubility coefficient of O2 in blood (mL O2/dL blood/mm Hg Pao2), and Pao2 is expressed in mm Hg.3 The main determinants of oxygen content of arterial blood are hemoglobin concentration and hemoglobin saturation (which is largely determined by Pao2).
Do2 (mL/min) is dependent on Cao2 (mL/L) and cardiac output (CO, L/min).3 Do2 =Cao2 ×CO The main determinants of oxygen delivery to the tissues are hemoglobin concentration, hemoglobin saturation (which is largely determined by Pao2), and cardiac output.
930
oxygen delivery to the tissues? What is hypoxia?
When can hypoxia occur in the absence of hypoxemia?
Hypoxia refers to the deficiency or ineffective use of oxygen within tissues of the body. Causes include stroke, myocardial infarction, claudication, and intestinal ischemia. Hypoxia cannot be measured directly in the same way that hypoxemia can. However, blood lactate concentration is often elevated in the setting of tissue hypoxia, producing lactic acidosis. Hypoxemia is the most common cause of hypoxia, but these conditions can occur independently.1,2 Hypoxia can occur in the absence of hypoxemia when any of the following actions are impaired: oxygen delivery (eg, low cardiac output, anemia, thrombus), oxygen binding to hemoglobin (eg, methemoglobinemia), and oxygen uptake or use by the tissues (eg, carbon monoxide poisoning, sepsis, cyanide poisoning).
What is the effect of Increased tissue oxygen extraction on the partial pressure of oxygen in arterial blood? What is the alveolar gas equation?
Conditions that reduce Do2 (eg, anemia, heart failure) trigger a compensatory increase in oxygen extraction by the tissues, resulting in abnormally low oxygen content of mixed venous blood entering the pulmonary capillary bed. In the setting of abnormal gas exchange (eg, pulmonary edema), the presence of low oxygen content of blood entering the pulmonary capillary bed will have a significant impact on the oxygen content of blood leaving the pulmonary capillary bed, worsening the degree of hypoxemia.3 Pao2 =(Fio2 ×[PB −]) −(Paco2/RQ) In the above formula, Pao2 is the partial pressure of alveolar oxygen (mm Hg), Fio2 is the fraction of inspired oxygen, PB is the barometric pressure (atmospheric pressure) (mm Hg), is water vapor pressure in the airways (mm Hg), Paco2 is the partial pressure of carbon dioxide in arterial blood (mm Hg), and RQ is the respiratory quotient. Several underlying causes of hypoxemia can be readily identified using this equation, as demonstrated in subsequent sections.
What is the fraction of inspired oxygen of room air? What is the barometric pressure at sea level? What is the water vapor pressure in the airways at 37°C? What is normal partial pressure of carbon dioxide in arterial blood? What is the respiratory quotient?
Based on the alveolar gas equation, what is the partial pressure of alveolar oxygen in a patient breathing room air with normal partial pressure of carbon dioxide in arterial blood? The causes of hypoxemia can be separated into which 2 general categories?
What is the A-a gradient?
Fio2 of room air is 21%.3 PB at sea level is 760 mm Hg.3 at 37°C is 47 mm Hg.3
Normal Paco2 is 40 mm Hg.3 The RQ is the ratio of carbon dioxide produced by the tissues to oxygen consumed by the tissues. It is dependent on the patient’s dietary fat, protein, and carbohydrate content. The average value for a balanced diet at resting state is 0.8 (range is 0.7-1.0).3 Pao2 =(0.21 ×[760 −47]) −(40/0.8) =∼100 mm Hg.
Hypoxemia can occur in the setting of normal A-a gradient or elevated A-a gradient.
The A-a gradient is the difference in partial pressure of oxygen between the alveolus (A) and arterial blood (a). Pao2 is calculated using the alveolar gas equation, whereas Pao2 is measured with a sample of arterial blood (arterial blood gas test) (Figure 46-4).
FIGURE 46-4 Gas exchange in the lung. The A-a gradient is the difference in partial pressure of oxygen between the alveolar space and arterial blood. (From Rhoades RA, Bell DR. Medical Physiology: Principles for Clinical Medicine. 5th ed. Philadelphia, PA: Wolters Kluwer; 2018.)
What is a normal A-a gradient? Normal A-a gradient increases with age and Fio2. The following table indicates the upper limits of normal A-a gradient at room air according to age:
Table 46-1 Age (years) 20 30 40 50
A-a gradient (mm Hg) 17 21 24 27
931
60 31 70 34 80 38 Normal A-a gradient increases 5 to 7 mm Hg for every 10% increase in Fio2.3,5
932
Hypoxemia Associated with Normal A-a Gradient
What are the 2 general causes of hypoxemia with normal A-a gradient?
Hypoxemia associated with normal A-a gradient can be caused by reduced partial pressure of inspired oxygen (Pio2) or hypoventilation.
These categories of hypoxemia are easily identified within the alveolar gas equation.
933
Hypoxemia Related to REDUCED Pio2
What are the 2 main causes of reduced partial pressure of inspired oxygen?
What is the A-a gradient in a patient breathing air with a fraction of inspired oxygen of 16% at normal barometric pressure, with a normal partial pressure of carbon dioxide in arterial blood, and a partial pressure of oxygen in arterial blood of 58 mm Hg? How does the hypoxemia related to reduced partial pressure of inspired oxygen respond to inhaled oxygen?
A reduction in either Fio2 or PB can result in reduced Pio2. Pao2 =(0.16 ×[760 −47]) −(40/0.8) =∼64 mm Hg. A-a gradient =64 −58 =6 mm Hg. This value is normal, as expected in a patient with hypoxemia driven purely by reduced Pio2. Decreased Pao2 related to reduced Pio2 improves with inhaled oxygen.6
What are the causes of hypoxemia related to reduced Pio2?
Firefighters are at Increased risk. A toddler passes out after playing with a large plastic bag. Aircraft cabins are pressurized for this reason.
What is the fraction of inspired oxygen in a patient with true oxygen saturation of hemoglobin in arterial blood of 50%, assuming an A-a gradient of 5 mm Hg and normal partial pressure of carbon dioxide in arterial blood? What are some causes of suffocation that can lead to hypoxemia? How is the partial pressure of alveolar oxygen affected by altitude?
Inhalation of polluted air from smoke. Suffocation. High altitude.
The Pao2 at which hemoglobin is 50% saturated is 27 mm Hg (see Figure 46-3). If the A-a gradient is 5 mm Hg, then the Pao2 must be 32 mm Hg. Using the alveolar gas equation, Fio2 =([40/0.8] +32)/(760 – 47) =0.115 (or 11.5% oxygen).
Causes of suffocation include enclosed and poorly ventilated spaces (eg, a child left in a car with the windows rolled up), drowning, and accidental suffocation (eg, a toddler with a plastic bag). At increasing altitudes, Fio2 remains constant at 0.21 (21%) but PB decreases, resulting in reduced Pio2. For example, at 8400 m above sea level, PB is approximately 272 mm Hg. In a person with compensatory hyperventilation and a Pco2 of 20 mm Hg, Pao2 =(0.21 ×[272 −47]) − (20/0.8) =∼22 mm Hg. One of the lowest recorded Pao2 values logged in a healthy individual was 19 mm Hg, which was recorded on Mount Everest at 8400 m (Figure 46-5).7
FIGURE 46-5 Relationship between altitude, barometric pressure (atmospheric pressure), and inspired oxygen. Note that at an altitude of 5000 m (∼16,000 ft), the highest at which humans reside, the inspired Pio2 is approximately half of the value at sea level. On the summit of Mount Everest, at 8850 m (∼29,030 ft), the inspired Pio2 is less than 30% of the value at sea level. (From West JB. The physiologic basis of high-altitude diseases. Ann Intern Med. 2004;141:789- 800.)
934
935
Hypoxemia Related to Hypoventilation
How does hypoventilation affect the partial pressure of carbon dioxide in arterial blood? What is the A-a gradient in a patient breathing room air at sea level, with an elevated partial pressure of carbon dioxide in arterial blood of 80 mm Hg, and a measured partial pressure of oygen in arterial blood of 45 mm Hg? How does the hypoxemia related to hypoventilation respond to inhaled oxygen?
Paco2 increases as a result of hypoventilation. Pao2 =(0.21 ×[760 −47]) −(80/0.8) =∼50 mm Hg. A-a - gradient =50 −45 =5 mm Hg. This value is normal, as expected in a patient with hypoxemia driven purely by hypoventilation. Decreased Pao2 related to hypoventilation generally improves with inhaled oxygen, but it is more effective to increase ventilation.6
What are the causes of hypoxemia related to hypoventilation?
A 24-year-old man is brought to the Drugs and toxins. emergency department with hypoventilation and hypoxemia and is noted to have track marks near the antecubital regions of his arms. Large neck circumference is a Obstructive sleep apnea (OSA). significant risk factor for this condition. This patient was described in the Obesity hypoventilation syndrome (OHS, or Pickwickian syndrome). Charles Dickens novel “The Posthumous Papers of the Pickwick Club.” A 26-year-old woman with asthma Severe airway obstruction related to asthma exacerbation. presents with obtundation and Paco2 of 90 mm Hg after being exposed to cat dander. A 34-year-old woman presents with Neuromuscular Weakness Related to Guillain-Barré syndrome. ascending paralysis complicated by shortness of breath and hypoxemia 2 weeks after a diarrheal illness. Abnormal curvature of the spine in Mechanical obstruction related to kyphoscoliosis. the coronal and sagittal planes. Hypoventilation as a compensatory Metabolic alkalosis. mechanism.
Which 2 main classes of substances, Narcotics and benzodiazepines are common causes of depressed respiratory drive. Naloxone may used both as prescription be given as an antidote for narcotic toxicity, whereas flumazenil is the antidote of choice for medications and drugs of abuse, benzodiazepine toxicity. cause hypoventilation by depressing the respiratory drive? What are the common symptoms of Symptoms of OSA include excessive daytime sleepiness, snoring, and morning headache. Weight obstructive sleep apnea? loss and nocturnal continuous positive airway pressure (CPAP) are the initial treatments of choice.8 How is obesity hypoventilation OHS is defined by the constellation of obesity (body mass index >30 kg/m2), daytime syndrome defined? hypoventilation (awake Paco2 >45 mm Hg), and sleep-disordered breathing in the absence of other causes of hypoventilation. Weight loss and treatment of coexistent OSA with CPAP are the initial strategies of choice.8 What are the main contributors to Hypoventilation in patients with acute exacerbation of COPD is related to worsening airflow hypoventilation in patients with obstruction from mucus or bronchoconstriction, combined with respiratory muscle fatigue. acute exacerbation of chronic Noninvasive treatment strategies include inhaled short-acting bronchodilators (eg, albuterol),
936
obstructive pulmonary disease (COPD)? What are some causes of neuromuscular weakness that can lead to hypoventilation?
What are the causes of mechanical obstruction that can lead to hypoventilation? What is the expected partial pressure of carbon dioxide in arterial blood in a patient with primary metabolic alkalosis and serum bicarbonate of 48 mEq/L?
glucocorticoids (eg, prednisone), antibiotics (eg, azithromycin), and noninvasive positive pressure ventilation (NPPV) via CPAP or bilevel positive airway pressure (BiPAP).8 Neuromuscular weakness can occur as a result of the following conditions: upper motor neuron disease (eg, spinal cord injury), anterior horn cell disease (eg, amyotrophic lateral sclerosis), peripheral neuropathy (eg, Guillain-Barré syndrome), disorders of the neuromuscular junction (eg, myasthenia gravis), muscular dystrophy (eg, Duchenne), myopathy (eg, polymyositis), and electrolyte disturbance (eg, hypophosphatemia). Causes of mechanical obstruction include obesity, ascites, and restrictive chest wall disorders (eg, kyphoscoliosis, ankylosing spondylitis, fibrothorax, multiple rib fractures).8 The following formula, which is predominantly based on serum bicarbonate () concentration, can be used to predict the Paco2 in the setting of a metabolic alkalosis: Expected Paco2 =0.7 ×([] −24) +40 ± 2. In this example, expected Paco2 =0.7 ×(48 −24) +40 ± 2 =57 ± 2 mm Hg.9
937
Hypoxemia Associated with Elevated A-a Gradient
What are the 4 general mechanisms Hypoxemia associated with elevated A-a gradient can occur as a result of dead space (a form of of hypoxemia related to elevated A-a ventilation-perfusion [V/Q] mismatch), physiologic shunt (a form of V/Q mismatch), impaired gradient? diffusion, or anatomic shunt.
Lung diseases often cause hypoxemia with elevated A-a gradient through a combination of these mechanisms (eg, emphysema is associated with Increased dead space, shunt, and impaired diffusion). However, most etiologies predominantly act through 1 mechanism in particular, and that principle will be used in this chapter to organize the framework.
What is the ventilation- perfusion ratio?
The V/Q ratio describes the relationship between ventilation of the alveoli and perfusion of the pulmonary capillaries. A perfect match between ventilation and perfusion (ie, V/Q of 1) is the reference point for defining normal and abnormal gas exchange in the lungs (Figure 46-6).3
FIGURE 46-6 Matching of ventilation and perfusion. Center: normal matching of ventilation and perfusion; left: perfusion without ventilation (ie, shunt); right: ventilation without perfusion (ie, dead space). (From Porth CM. Essentials of Pathophysiology: Concepts of Altered Health States. 4th ed. Philadelphia, PA: Wolters Kluwer; 2015.)
938
Hypoxemia Related to Increased Dead Space
What is dead space?
Dead space occurs when there is excess alveolar ventilation relative to pulmonary capillary perfusion (ie, V/Q >1). Anatomic dead space is normal and refers to parts of the airway that are not normally involved in gas exchange (eg, the trachea); it accounts for 20% to 30% of total ventilation (roughly 1 mL per pound of body weight). Alveolar dead space is abnormal and refers to areas of the lung that are normally involved in gas exchange. The combination of anatomic and alveolar dead space is referred to as physiologic dead space (Figure 46-7). Increased physiologic dead space can result in both hypoxemia and hypercapnia. Because of compensatory hyperventilation, hypercapnia does not occur until dead space is >50% of total lung volume.3
FIGURE 46-7 A, Absence of blood flow to an alveolar region. B, Reduced blood flow to an alveolar region. In both cases, a
What local compensatory mechanism occurs to offset the effects of pathologic dead space? How does the hypoxemia related to dead space respond to inhaled oxygen?
portion of alveolar air does not participate in gas exchange and constitutes alveolar dead space volume. Physiologic dead space is the sum of alveolar dead space plus anatomic dead space. (From Rhoades RA, Bell DR. Medical Physiology: Principles for Clinical Medicine. 5th ed. Philadelphia, PA: Wolters Kluwer; 2018.)
In the setting of Increased dead space, compensatory bronchoconstriction acts to normalize the V/Q ratio.6,10
Decreased Pao2 related to dead space generally improves with inhaled oxygen.6
What are the causes of Increased physiologic dead space?
Acute right-sided heart strain in a patient with an erythematous and edematous lower extremity. Hyperinflation, bleb formation, and Increased lung compliance. A global decrease in pulmonary capillary perfusion. Listen for a loud (and sometimes palpable) pulmonic component of the second heart sound (P2). Often associated with small vessel vasculitis and glomerulonephritis. Iatrogenic.
Pulmonary embolism (PE).
Emphysema. Hypotension. Pulmonary hypertension.
Pulmonary capillaritis (eg, granulomatosis with polyangiitis [GPA, or Wegener’s granulomatosis]). Positive pressure ventilation (PPV).
939
What are the principal mechanisms In the setting of PE, there is a shift in blood flow to the unaffected parts of the lungs, causing a of hypoxemia related to pulmonary relative V/Q mismatch in the form of physiologic shunt, resulting in hypoxemia. In some patients, embolism? Increased right-sided intracardiac pressures related to the PE lead to a right-to-left intracardiac shunt through the foramen ovale or a different atrial septal defect.11,12 What additional mechanism of Emphysema is associated with impaired gas transfer. hypoxemia is often identified on pulmonary function testing in patients with emphysema? What cause of hypotension should be Septic shock is characterized by normal or low central venous pressure and Decreased systemic suspected in a patient with fever, vascular resistance, and should be suspected in patients who present with fever along with these normal central venous pressure, and hemodynamic changes. Decreased systemic vascular resistance? What are the additional mechanisms In addition to increasing dead space, pulmonary hypertension causes hypoxemia through a of hypoxemia related to pulmonary variety of mechanisms, including impaired diffusion and anatomic shunt (Increased right-sided hypertension? heart pressures can force blood through the foramen ovale). Additionally, Decreased cardiac output related to cor pulmonale can result in low mixed venous Po2, contributing to hypoxemia.13 What condition can occur in patients Diffuse alveolar hemorrhage (DAH) is a severe complication of pulmonary capillaritis that can with pulmonary capillaritis that result in massive hemoptysis and physiologic shunt. results in hemoptysis and physiologic shunt? Why does positive pressure Physiologic dead space is Increased by PPV for the following 2 reasons: (1) increase in alveolar ventilation cause Increased pressure with resultant overdistention and (2) reduced venous return to the right side of the heart, physiologic dead space? which causes a decrease in pulmonary blood flow.3
940
Hypoxemia Related to Physiologic Shunt
What is a physiologic shunt?
How does the hypoxemia related to physiologic shunt respond to inhaled oxygen?
Physiologic shunting occurs when there is excess pulmonary capillary perfusion relative to alveolar ventilation (ie, V/Q <1) (see Figure 46-6). When there is partial gas exchange, it is referred to as “venous admixture.” When there is total absence of any gas exchange, it is referred to as a “true shunt,” which is equivalent to an anatomic shunt.3 The response to inhaled oxygen in the setting of physiologic shunt depends on the degree of shunt present. As the shunt fraction increases from normal (<10%) to 50%, there is an incremental decrease in response to inhaled oxygen. At shunt fractions >50%, Pao2 is independent of the fraction of inspired oxygen, behaving like a true or anatomic shunt. Understanding this concept can prevent iatrogenic oxygen toxicity in these patients.3
What are the causes of physiologic shunt?
Pus in the alveolar space. Fluid in the alveolar space. Alveolar collapse. Wheezing is usually appreciated on auscultation of the chest. Blood in the alveolar space. A patient with cirrhosis experiences dyspnea that worsens in the upright position and improves when lying supine.
In patients with acquired immunodeficiency syndrome who develop pneumonia from Pneumocystis jirovecii (PJP), why is it important to evaluate for hypoxemia? What are the cardiogenic causes of pulmonary edema? What are the causes of noncardiogenic pulmonary edema?
What are the key features of acute respiratory distress syndrome?
What are the causes of acute respiratory distress syndrome?
How does the distribution of fluid on chest imaging differ between cardiogenic and noncardiogenic pulmonary edema? What lung conditions can predispose to mucous plugging of the small airways? Which clue on the chest radiograph can be used to determine if complete hemithorax opacification
Pneumonia. Pulmonary edema. Atelectasis. Airway constriction. Diffuse alveolar hemorrhage. Hepatopulmonary syndrome.
In patients with moderate-to-severe PJP pneumonia (defined as Pao2 <70 mm Hg or A-a gradient of
35), early treatment with glucocorticoids (within 72 hours after initiation of antipneumocystis therapy) is associated with a significant reduction in mortality.14
Left-sided heart failure is by far the most common cardiogenic cause of pulmonary edema; others include mitral stenosis and mitral regurgitation with flail posterior leaflet (which results in focal right upper lobe pulmonary edema). Causes of noncardiogenic pulmonary edema include acute respiratory distress syndrome (ARDS, which can be caused by a variety of underlying conditions), drug toxicity (eg, narcotic overdose), inhalation injury, high altitude pulmonary edema, neurogenic pulmonary edema, reexpansion pulmonary edema (ie, pulmonary edema following large-volume thoracentesis), and reperfusion pulmonary edema.15 ARDS is characterized by the acute development (within 1 week) of a respiratory illness associated with severe hypoxemia (Pao2/Fio2 ratio of ≤300) and bilateral opacities on chest imaging consistent with pulmonary edema that cannot be explained by cardiac disease.15 Bacterial and viral pneumonias are the most common causes of ARDS; others include sepsis, aspiration of gastric contents, acute pancreatitis, drug reactions (eg, methotrexate), inhalation of toxic fumes or particles (eg, massive smoke inhalation), and nonthoracic trauma.16 Cardiogenic pulmonary edema tends to occur in a dependent distribution.
Mucous plugging of the small airways commonly occurs in the setting of COPD, asthma, cystic fibrosis, and bronchiectasis.
Tracheal deviation toward an opacified hemithorax suggests volume loss in that area (eg, atelectasis from a mucous plug); tracheal deviation away from an opacified hemithorax suggests a volume-occupying process (eg, pleural effusion).
941
is related to a mucous plug of a large airway or a pleural effusion? What condition associated with Reexpansion pulmonary edema is an uncommon complication of thoracentesis that occurs within hypoxemia can develop in patients 24 hours of the procedure (usually within 1-2 hours). Risk factors include younger age, duration of who undergo thoracentesis with lung collapse >72 hours, removal of large volumes of pleural fluid (>1500 mL), and rapid large-volume drainage? reexpansion. The vast majority of patients recover within 5 to 7 days.17 Obliterative bronchiolitis is Patients who have undergone either lung or hematopoietic stem cell transplantation are at risk for associated with which types of developing obliterative bronchiolitis. It is more common in lung transplant recipients, where it transplant procedures? develops in most long-term survivors. Other associated conditions include autoimmune diseases (eg, rheumatoid arthritis) and exposure to inhalational toxins (eg, diacetyl).18 What is the key difference between Unlike with COPD, airway obstruction related to asthma demonstrates reversibility. Hypoxemia asthma and chronic obstructive can be a feature of severe asthma and COPD exacerbations, caused by shunt from airway pulmonary disease on pulmonary constriction as well as hypoventilation. Pharmacologic management includes inhaled short-acting function testing? bronchodilators, glucocorticoids, and antibiotics in some cases. Noninvasive and invasive ventilation techniques may be necessary in some patients.8 What is the treatment for diffuse Most cases of DAH are related to pulmonary capillaritis (eg, GPA). Such cases may be responsive alveolar hemorrhage? to immunosuppressive therapy (eg, glucocorticoids, cyclophosphamide, rituximab). Other etiologies of or contributors to DAH, such as infection and medications, should be specifically addressed.19 What are the main clinical Dyspnea is the main symptom of hepatopulmonary syndrome and can be triggered by moving manifestations of hepatopulmonary from a supine to an upright position (ie, platypnea). In addition to physical findings of cirrhosis syndrome? (eg, spider angiomas), there may be digital clubbing, cyanosis, and orthodeoxia (defined by a decrease in Pao2 by ≥5% or by ≥4 mm Hg when moving from a supine to an upright position).20
942
Hypoxemia Related to Impaired Diffusion Capacity
What is diffusion capacity?
Diffusion capacity describes the efficiency of gas transfer from the alveolar space to the pulmonary capillaries, which is principally dependent on the integrity of the alveolar-capillary membrane (Figure 46-8). In the laboratory, it is evaluated by measuring the diffusing capacity for carbon monoxide (Dlco).
FIGURE 46-8 Gas exchange in the lung. Oxygen and carbon dioxide move across the alveolar-capillary membrane via Fick’s laws of diffusion. (From Kraemer WJ, Fleck SJ, Deschenes MR. Exercise Physiology Integrating Theory and Application. 2nd ed. Philadelphia, PA: Wolters Kluwer; 2016.)
Why does CO2 is approximately 20 times more soluble in water than O2, making it significantly less likely to be affected by impaired diffusion result impaired diffusion. Furthermore, hyperventilation is often induced by hypoxemia, leading to a reduction in Paco2.6 in hypoxemia but not hypercapnia? What is the Hypoxemia is significantly worsened during exercise in patients with impaired diffusion. This is principally related to response to an increase in cardiac output during exercise, which reduces pulmonary capillary transit time, further impairing exercise in oxygen diffusion. Moreover, tissue oxygen extraction increases during exercise, which results in Decreased mixed patients with venous oxygen content entering the pulmonary capillaries. The 6-minute walk test and cardiopulmonary exercise testing are impaired diffusion? useful in the evaluation of patients with impaired diffusion.6 How does the Decreased Pa related to impaired diffusion generally improves with inhaled oxygen.6 hypoxemia o 2 related to impaired diffusion respond to inhaled oxygen?
What are the causes of impaired diffusion?
A 54-year-old woman with Interstitial lung disease (ILD). dermatomyositis develops dyspnea and hypoxemia, and is found to have fine, end-inspiratory rales on auscultation of the lungs.
What are the characteristic Pulmonary function testing in a patient with ILD typically reveals a restrictive pattern (low forced vital findings of interstitial lung capacity [FVC] and forced expiratory volume in 1 second [FEV ] with normal FEV /FVC) in combination 1 1 disease on pulmonary with impaired Dlco. Impaired diffusion may be the only abnormality in early ILD. The 6-minute walk test function testing? (which measures distance walked and oxygen desaturation) correlates with severity and can be prognostic in some forms of ILD. Interstitial lung disease is discussed in depth in chapter 47, Interstitial Lung Disease.21 Other conditions can be associated with impaired diffusion, such as
943
pulmonary hypertension and emphysema.
944
Hypoxemia Related to Anatomic Shunt
What is an anatomic shunt? An anatomic shunt occurs when venous blood completely bypasses the pulmonary capillaries and enters the systemic circulation (Figure 46-9). A normal anatomic shunt occurs when the bronchial veins drain directly into the pulmonary veins (which amounts to 2%-3% of cardiac output).6
FIGURE 46-9 Illustration of an anatomic shunt, showing venous blood bypassing the alveoli and entering
How does the hypoxemia related to anatomic shunt respond to inhaled oxygen?
systemic circulation (arrows). (Adapted from Rhoades RA, Bell DR. Medical Physiology: Principles for Clinical Medicine. 5th ed. Philadelphia, PA: Wolters Kluwer; 2018.)
The Pao2 response to inhaled oxygen in the setting of an anatomic shunt is independent of the fraction of inspired oxygen. This information can be useful in identifying the presence of an anatomic shunt.3
What are the causes of anatomic shunt?
Echocardiography with agitated saline contrast results in the immediate appearance of bubbles in the left side of the heart. Echocardiography with agitated saline contrast results in the delayed appearance of bubbles in the left side of the heart.
What condition can occur with sudden-onset in patients with right-to-left intracardiac shunt, sometimes with devastating outcomes? What is the relationship between pulmonary arteriovenous malformation and hereditary hemorrhagic telangiectasia (HHT, or Osler-Weber-Rendu syndrome)?
Intracardiac shunt.
Pulmonary arteriovenous malformation.
Patients with right-to-left shunt are at risk for experiencing paradoxical embolism to critical organs such as the brain and kidneys.
The majority of cases of pulmonary AVM are congenital and associated with HHT; however, only a minority of patients with HHT develop pulmonary AVMs. Symptoms usually present between 30 and 60 years of age. An example of acquired pulmonary AVMs occurs in some patients with hepatopulmonary syndrome.22
945 Case Summary foA 51-year-old man with cirrhosis presents with dyspnea and is What is the most likely cause of hypoxemia in this patient? Hepatopulmonary syndrome. und to have positional hypoxemia.
946
Bonus Questions
What is the A-a gradient in this case? Pao =(0.21 ×[760 −47]) −(32/0.8) =110 mm Hg. A-a gradient =110 −56 =54 mm Hg. The upper limit of normal A-a gradient in a 50-year-old 2 patient is approximately 27 mm Hg (see Table 46-1). The A-a gradient is therefore elevated, indicating that the mechanism of hypoxemia 2 (Recall ABG showed Pao of 56 mm Hg and Paco2 of 32 mm Hg.) this case involves 1 or more of the following: Increased dead space, physiologic shunt, impaired diffusion capacity, and anatomic shunt. in What finding is present inithe photograph The photograph of the patient’s hand in this case (see Figure 46-1) demonstrates cyanosis and digital clubbing (one of the oldest signs in of the patient’s hand in th s case? medicine originally described by Hippocrates in a patient with empyema). 23 What causes digital clubbing? Digital clubbing occurs when circulatingtmegakaryocytes, normally trapped in the lung, bypass the filterithrough a right-to-left shunt and become lodged in the peripheral vascula ure of the digits. There, the megakaryocytes release platelet-der ved growth factor and vascular endothelial growth factor, which lead to the connective tissue changes found in clubbed digits. 23 What would be the expected findings if a The chest radiograph in patients with pure hepatopulmonary syndrome is typically normal. Patients with cirrhosis could have abnormalities chest radiograph were performed in this related to other pathophysiologic processes (eg, pleural effusion [hepatic hydrothorax]). case? related to hepatopulmonary syndrome? pulmonary capillaries to 15tto 100 µm (normal is <8-15 µm), whichiincreases perfusion relative to ventilation (ie, physiologic shunt). In n of What are the mechanisms of hypoxemia Hypoxemia related to hepa opulmonary syndrome is caused by physiologic and/or anatomic shunt. The main mechanism is the dilatio addition, pulmonary arteriovenous communications can develop ( e, anatomic shunt). 20 How is hepatopulmonary syndrome Transthoracic echocardiography withlagitated saline contrast is the most practical method for diagnosingthepatopulmonary syndrome. e diagnosed? Normally, the microbubbles are unab e to pass through the pulmonary capillary bed; however, when there is abnormal dilation of the capillary bed, with or without pulmonary AVMs, the bubbles are able to pass through, resulting in a posi ive study (see Figure 46-2). Th administration of 100% oxygen in the pulmonary function laboratory can be used to calculate the shunt fraction. 20 What is the prognosis of me? Liver transplantation is the only effective treatment for hepatopulmonary syndrome. Without it, median survival is aroundf24 months and 5- 2 hepatopulmonary syndro year survival rates are approximately 25%. Survival is significantly shorter in patients with a Pao <50 mm Hg at the time o diagnosis. Identifying hepatopulmonary syndrome is important, as it may improve a patient’s wait list status and candidacy for liver transplantation.20 What additional pulmonary vascular Portopulmonary hypertension, which is defined as pulmonary arterial hypertension in association with portal hypertension of any etiology, condition that sometimes develops in should be suspected in patients with cirrhosis who develop hypoxemia. The diagnosis can be established with hemodynamic measurements patients with portal hypertension could be from right heart catheterization.24 case? contributing to the hypoxemia in this
947 Key Points
Hypoxemia is defined as low Pao (<80 mm Hg). 2 Hypoxia refers to the deficiency or ineffective use of oxygen Hypoxemia is the most common cause of hypoxia, but the two can within tissues of the body. occur independently. Hypoxia can occur in the absence of hypoxemia when any of the
following actions are impaired: oxygen delivery (eg, low cardiac ), Pulse oximetry is a valuable noninvasive tool for the evaluation of Arterial blood gas analysis allows for definitive evaluation of output), oxygen binding to hemoglobin (eg, methemoglobinemia and oxygen uptake or use by the tissues (eg, sepsis). hypoxemia. hypoxemia. Hemoglobin saturations of 50% and 90% correspond to Pao values 2 of 27 and 60 mm Hg, respectively. io ×[P −]) −(Paco /RQ), allows The alveolar gas equation, P ao =(F 2 2 B 2 for calculation of the A-a gradient, which is the first step in Normal A-a gradient increases with age (eg, 17 mm Hg at age evaluating the cause of hypoxemia. 20 years, 38 mm Hg at age 80 years). Hypoxemia associated with normal A-a gradient can be caused by io2 reduced P or hypoventilation. Hypoxemia associated with elevated A-a gradient can be caused
by dead space, physiologic shunt, impaired diffusion, or anatomic Lung diseases often cause hypoxemia through a combination of perfusion (eg, PE); physiologic shunt occurs when thereiis excess Impaired diffusion is the abnormal transfer of gas across the shunt. mechanisms. Dead space occurs when there is excess ventilation relat ve to perfusion relative to ventilation (eg, pneumonia). alveolar-capillary membrane (eg, ILD). Anatomic shunt occurs when venous blood comp etely bypasses the pulmonary capillaries and enters the systemiclcirculation (eg, intracardiac shunt).
948
949
References 1. Hypoxemia vs. hypoxia. N Engl J Med. 1966;274(16):908-909. 2. Myers JA, Millikan KW, Saclarides TJ,.eds. Common Surgical Diseases: An Algorithmic 3. Marino PL. The ICU Book. 3rd ed. Philadelphia, PA: Lippincott Williams & Wilkins – A Approach to Problem Solving. 2nd ed New York, NY: Springer New York; 2008. Wolters Kluwer business; 2007.
-
Berne RML, Levy MN. Physiology. 4th ed. St. Louis, Missouri: Mosby, Inc.; 1998. 5. Morris AH, ed. ClinicallPulmonary Function: A Manual of Uniform Laboratory 6. Sarkar M, Niranjan N, Banyal PK. Mechanisms of hypoxemia. Lung India. Procedures. 2nd ed. Sa t Lake City: InterMountain Thoracic Society; 1984. 2017;34(1):47-60.
-
Brown JPG, Michael PW. Humans at altitude: physiology and pathophysiology. Contin Educ Anaesth Crit Care Pain. 2013;13(1):17-22.
-
Chebbo A, Tfaili A, Jones SF. Hypoventilation syndromes. Med Clin North Am. 2011;95(6):1189-1202.
-
Berend K, de.Vries AP, Gans RO.;Physiological approach to assessment of acid-base 10. Gurewich V, Thomas D, Stein M,.Wessler:S. Bronchoconstriction in the presence of 11. Jardin F, Gurdjian F, DesfondsiP, Fouilladieu JL, Margairaz A. Hemodynamic factors disturbances N Engl J Med. 2014 371(15):1434-1445. pulmonary embolism. Circulation 1963;27 339-345. influencing arterial hypoxemia n massive pulmonary embolism with circulatory failure. Circulation. 1979;59(5):909-912. . Kasper W, Geibel A, Tiede N, Just H. Patent foramen ovale in patients with
12haemodynamically significant pulmonary embolism. Lancet. 1992;340(8819):561-564. 13. Vodoz JF, Cottin V, Glerant JC, et al. Right-to-left shunt with hypoxemia in pulmonary hypertension. BMC Cardiovasc Disord. 2009;9:15. . Consensus statement on the use of corticosteroids as adjunctive therapy for pneumocystis pneumonia in the acquired immunodeficiency syndrome. The N tion
14Institutes of Health-University of California Expert Panel for Corticosteroids asa500-al Adjunctive Therapy for Pneumocystis Pneumonia. N Engl J Med. 1990;323(21):1 1504.
-
Ferguson ND, Fan E, Camporota L, et al. The BerlinIdefinition of ARDS: an expanded rationale, justification, and supplementary material. ntensive Care Med. 2012;38(10):1573-1582.
-
Matthay MA, Ware LB, Zimmerman GA. The acute respiratory distress syndrome. J Clin Invest. 2012;122(8):2731-2740.
-
Kasmani R, Irani F, Okoli K, Mahajan V. Re-expansion pulmonary edema following thoracentesis. CMAJ. 2010;182(18):2000-2002.
-
Barker AF, Bergeron A, Rom WN, Hertz MI. Obliterative bronchiolitis. N Engl J Med. 2014;370(19):1820-1828.
-
Park MS. Diffuse alveolar hemorrhage. Tuberc Respir Dis (Seoul). 2013;74(4):151-162. 20. Rodriguez-Roisin R, Krowka MJ. Hepatopulmonary syndrome–a liver-induced lung vascular disorder. N Engl J Med. 2008;358(22):2378-2387.
-
Wallis A, Spinks K. The diagnosis and management of interstitial lung diseases. BMJ. 2015;350:h2072.
950
-
Gossage JR, Kanj G. Pulmonary arteriovenous malformations. A state of the art review. Am J Respir Crit Care Med. 1998;158(2):643-661.
-
Sarkar M, Mahesh DM, Madabhavi I. Digital clubbing. Lung India. 2012;29(4):354-362. 24. Porres-Aguilar M, Altamirano;JT, Torre-Delgadillo A, Charlton:MR, Duarte-Rojo A. Portopulmonary hypertension and hepatopulmonary syndrome a clinician-oriented overview. Eur Respir Rev. 2012 21(125):223-233.
951
CHAPTER 47
952
Interstitial Lung Disease
953 Case: A 58-year-old man with a tight skin A 58-year-old man presents to the emergency department with
difficulty breathing. He reports that he was healthy up until 10 years to cold environments. A few years later, he developed progressive ago when he noticed that his fingers would turn white when exposed skin tightening and Decreased mobility of his hands, becoming dependent on his wife for help with most tasks. He lives in a rura town and has not previously sought medical care Over the pas year, he developed a dry cough and difficulty catching.his breath; hetislnow gastroesophageal reflux for which the patient takes over-the-counter short of breath at rest. Review of systems is notable for medication. peHeart rate is 104 beats per minute, and respiratory rate is 26 breaths r minute. Hemoglobin oxygen saturation by pulse oximetry (Sp o 2 ) is 86% on room air (using an earlobe probe). Jugular venous pressure is decrescendo diastolic murmur over the second intercostal space along fine end-inspiratory rales most pronounced at the bases oftthe lungs. Figure 47-2A (coronal view) and Figure 47-2B (cross-sectional view of 2 12 cm H O. Cardiac auscultation reveals a loud P2 and an early the left sternal border that is augmented with inspiration. There are The patient’s hands are shown in Figure 47-1. High-resolu ion computed tomography (HRCT) imaging of the chest is shown in the lower lobes).
FIGURE 47-1
FIGURE 47-2
What is the most likely diagnosis in this patient?
954
What is interstitial lung disease (ILD)?
ILD describes a heterogeneous group of diseases that affect the lung parenchyma, including the alveoli, pulmonary capillaries, and interstitial spaces, producing characteristic clinical, physiologic, imaging, and histologic manifestations (Figure 47-3).1,2
FIGURE 47-3 Microscopic elements of the pulmonary parenchyma. (From Ross MH, Pawlina W. Histology: A Text and Atlas with Correlated Cell.and Molecular Biology. 7th ed. Philadelphia, PA: Wolters Kluwer Health; 2016 )
What are the The most common symptoms of ILD include dyspnea, dry cough, fatigue, and weight loss. Additional symptoms symptoms of may accompany specific etiologies of ILD (eg, dry eyes in Sjögren’s syndrome). Symptom onset and disease course interstitial lung can be acute and rapidly progressive or insidious depending on the underlying etiology, and variability often exists disease? between patients with the same etiology.2 What are the Bilateral fine end-inspiratory rales are the hallmark of ILD. Other pulmonary findings may include pleural rubs and physical findings of inspiratory squeaks or squawks. Extrapulmonary findings may include digital clubbing and signs of pulmonary interstitial lung hypertension (eg, loud P2). Additional findings may be associated with specific etiologies of ILD (eg, Gottron’s disease? papules in dermatomyositis).2 What is the role of Conventional chest radiography is neither sensitive nor specific for the diagnosis of ILD. However, findings may conventional include Decreased lung volumes and Increased reticulonodular markings. Particular etiologies of ILD may be radiography in the suggested by the pattern of changes (eg, upper lobe predominance in sarcoidosis) or associated findings (eg, pleural evaluation of plaques in asbestosis). The chest radiograph can also be helpful in evaluating for alternative or additional sources of interstitial lung symptoms such as pulmonary edema or pleural effusions.2 disease? What is the role of HRCT imaging without contrast should be performed when ILD is suspected. The 2 main patterns of ILD on HRCT CT imaging in the imaging are usual interstitial pneumonia (UIP) and nonspecific interstitial pneumonia (NSIP).2 evaluation of interstitial lung disease? What are the The UIP pattern of ILD on HRCT imaging includes lower lung–predominant subpleural reticulations, traction characteristics of bronchiectasis, honeycombing (ie, cystic dilation of distal bronchioles), and minimal ground glass opacities.2 usual interstitial pneumonia on high-resolution CT imaging? What are the The NSIP pattern of ILD on HRCT imaging includes lower lung– predominant subpleural ground glass characteristics of opacification and reticulations and traction bronchiectasis in the absence of honeycombing.2 nonspecific interstitial pneumonia on high-resolution CT imaging? What are the On pulmonary function testing, ILD typically manifests a restrictive pattern (ie, low forced vital capacity [FVC] and findings of forced expiratory volume in 1 second [FEV ] with normal FEV /FVC) in combination with impaired gas transfer 1 1 interstitial lung testing (ie, impaired diffusing capacity for carbon monoxide [Dlco]). Impaired diffusion may be the only abnormality disease on pulmonary function in early ILD. The 6-minute walk test (which measures distance walked and oxygen desaturation) correlates with testing? severity and can be prognostic in some forms of ILD.2 What is the role of Bronchoscopy can be helpful in excluding infection in patients suspected of having ILD. In addition, bronchoscopy in the bronchoalveolar lavage (BAL) can generate cell differentials that may be supportive of specific etiologies of ILD (eg, evaluation of marked lymphocytic predominance in hypersensitivity pneumonitis). Endobronchial or transbronchial biopsies can interstitial lung occasionally establish the diagnosis and even the type of ILD (eg, sarcoidosis) but are generally too small to disease? diagnose most cases.2 What are the Surgical lung biopsy can be helpful in identifying ILD and specific subtypes. As with HRCT imaging, there are 2 histologic findings important histologic patterns of ILD: UIP and NSIP. Lung biopsy can be risky, particularly in older patients: open of interstitial lung lung biopsy is associated with a mortality of approximately 4% at 30 days, whereas video-assisted thoracoscopic disease? surgery (VATS) biopsy is associated with a mortality of about 2% at 30 days.2 What are the 2 ILD can be idiopathic or secondary to other conditions. general categories of interstitial lung disease?
955
Some secondary forms of ILD present with the imaging and histologic patterns of particular types of idiopathic ILD. In general, secondary ILD is more common than idiopathic ILD.
956
Idiopathic Interstitial Lung Disease
What is idiopathic interstitial lung disease?
The idiopathic forms of ILD present with characteristic clinical, imaging, and histologic patterns but are not related to known or identifiable systemic diseases or exposures.
What are the idiopathic forms of interstitial lung disease?
A previously healthy 68-year-old man develops slowly progressive dyspnea and dry cough over the course of a year and has fine end-inspiratory rales and digital clubbing on examination and a UIP pattern on HRCT imaging. The second main morphologic pattern of ILD.
Idiopathic pulmonary fibrosis (IPF). Because IPF is the clinical correlate of the morphologic pattern of UIP, the terms are often paired (ie, IPF/UIP). NSIP.
A 55-year-old woman presents to the emergency department with dyspnea, nonproductive cough, and fever for the third time in the past month despite several rounds of antibiotics and is found to again have a lung consolidation, but in a different location compared with prior episodes. Diffuse interstitial infiltration with lymphocytes, plasma cells, and macrophages, often associated with peribronchiolar reactive lymphoid follicles. The only idiopathic form of ILD with acute-onset symptoms. A 45-year-old woman with slowly progressive dyspnea and cough over a period of several months is found to have peripheral eosinophilia.
What are the clinical characteristics of idiopathic pulmonary fibrosis?
What are the clinical characteristics of nonspecific interstitial pneumonia?
What are the imaging features of cryptogenic organizing pneumonia?
What are the clinical characteristics of lymphoid interstitial pneumonia?
What are the clinical characteristics of acute interstitial pneumonia?
Cryptogenic organizing pneumonia (COP, or bronchiolitis obliterans with organizing pneumonia).
Lymphoid interstitial pneumonia (LIP).
Acute interstitial pneumonia (AIP, or Hamman-Rich syndrome). Chronic eosinophilic pneumonia (CEP).
IPF is the most common type of ILD, with a prevalence in the industrialized world of around 65 cases per 100,000 persons. It typically occurs in patients older than 50 years with insidious onset and gradual progression over years. It is eventually fatal in most cases, but certain therapies may slow progression, including tyrosine kinase inhibitors (eg, nintedanib), antifibrotics (eg, pirfenidone), and supplemental oxygen. Other therapies such as pulmonary rehabilitation and treatment for asymptomatic gastroesophageal reflux disease may also be beneficial. Lung transplantation should be considered for appropriate patients. Acute exacerbations of IPF occur in some patients and may be responsive to glucocorticoids.3,4 The onset of NSIP usually occurs between ages 40 and 50 years—about 10 years before the onset in the typical patient with IPF. Symptoms are similar to but milder than IPF. Treatment includes glucocorticoids and other immunosuppressive agents, and most patients stabilize or improve. Compared with IPF, prognosis of NSIP is better.5 The characteristic findings of COP on HRCT imaging are lung opacities, varying from ground glass to consolidation, located in a peripheral or peribronchial distribution, and most frequently involving the lower lobes. The opacities range in size from a few centimeters to an entire lobe and tend to be migratory on repeat imaging.5 LIP is more common in women and typically presents in the fifth decade of life. There is insidious progression of symptoms, including dyspnea and cough, over a period of 3 years or more. Idiopathic LIP is rare; it presents more commonly as a result of a secondary condition (eg, Sjögren’s syndrome, acquired immunodeficiency syndrome).5
Patients with AIP present at a mean age of 50 years, and symptoms typically develop within a 3-week time period, usually with an antecedent viral-like illness. Treatment is largely supportive, but glucocorticoids may be effective early in the course of the disease. Mortality rate is high, and
957
Which distinctive radiographic On chest imaging, peripheral airspace consolidation often referred to as the “photographic feature is associated with chronic negative shadow of pulmonary edema” may be seen in cases of CEP.5 eosSome secondary forms of ILD present with the morphologic patterns of the inophilic pneumonia? idiopathic forms of ILD (eg, ILD associated with Sjögren’s syndrome often presents in an LIP pattern). These secondary conditions must be ruled out before a diagnosis of idiopathic ILD is made.
958
Secondary Causes of Interstitial Lung Disease
What are the 2 general causes of secondary interstitial lung disease?
What general types of exposures can lead to interstitial lung disease?
ILD can occur as a result of exposure or systemic disease.
The causes of exposure-related ILD can be separated into the following subcategories: iatrogenic, hypersensitivity, pneumoconiosis, and other.
959
Interstitial Lung Disease Related to Iatrogenic Exposure
What iatrogenic exposures are associated with interstitial lung disease?
Sometimes it is not Medication. the underlying systemic disease that leads to interstitial lung disease but rather the treatment used. Look for tattoo History of thoracic external beam radiation therapy. marks on the chest.
What medications are associated with interstitial lung disease? How often does thoracic external beam radiation therapy result in pulmonary toxicity?
Numerous drugs are associated with ILD, including drugs for rheumatic disorders (eg, methotrexate), chemotherapeutic agents (eg, bleomycin), biological modifiers (eg, etanercept), selective serotonin reuptake inhibitors (eg, fluoxetine), antibiotics (eg, nitrofurantoin), and cardiac agents (eg, amiodarone). In some cases, drug withdrawal is followed by complete resolution of ILD.2 The occurrence and severity of lung disease after external beam radiation therapy is dependent on several factors, including dose delivered and volume of lung irradiated. Symptomatic radiation-induced pneumonitis typically develops 1 to 3 months after therapy. Some patients experience resolution of pneumonitis while others go on to develop pulmonary fibrosis. In some cases, pulmonary fibrosis develops in the absence of an acute phase. Fibrosis typically develops between 6 and 24 months post-treatment, achieving stability after 2 years. The radiographic pattern of disease tends to conform to the irradiation field, manifesting a variety of shapes (eg, rectangular).6
960
Interstitial Lung Disease Related to Hypersensitivity Pneumonitis
What is hypersensitivity pneumonitis (HP)? How common is hypersensitivity pneumonitis? What are the main classes of antigens that cause hypersensitivity pneumonitis? What is the clinical course of hypersensitivity pneumonitis? What are the clinical characteristics of acute hypersensitivity pneumonitis? What are the clinical characteristics of subacute hypersensitivity pneumonitis? What are the clinical characteristics of chronic hypersensitivity pneumonitis? What is the role of serologic testing in the evaluation of hypersensitivity pneumonitis? What are the radiographic findings of hypersensitivity pneumonitis? What are the findings of chronic hypersensitivity pneumonitis on high- resolution CT imaging? What is the role of bronchoscopy in the evaluation of hypersensitivity pneumonitis? What is the treatment for hypersensitivity pneumonitis?
HP (ie, extrinsic allergic alveolitis) describes a syndrome related to repeated inhalation of and sensitization to aerosolized antigens in susceptible hosts, resulting in inflammatory changes in the lung parenchyma and airways. There is usually a history of exposure with compatible clinical, imaging, and histologic findings.7 HP accounts for up to 15% of all cases of ILD in the industrialized world. However, many cases are unrecognized or mischaracterized, so the true prevalence may be higher.7 The main classes of antigens associated with HP include bacterial antigens, fungal antigens, animal or bird proteins, insect proteins, and chemicals. Patients may be exposed to these antigens in a variety of environments, including at home and in the workplace.7,8 The course of HP is highly variable, with some cases manifesting as an acute illness that resolves with no long-term sequelae and other cases manifesting with chronic pulmonary fibrosis. There is considerable overlap between the acute, subacute, and chronic forms of the disease (eg, a patient with chronic HP may have acute exacerbations with antigen exposure), and clinical criteria to distinguish the subtypes are lacking.7,8 The symptoms of acute HP typically occur within a few hours of antigen exposure and resemble an influenza-like illness. Wheezing is often present on examination. Acute HP is typical with thermophilic actinomycete species or fungal antigen exposure. Patients generally begin to improve over hours to days. Recurrences on antigen reexposure are common. Acute HP is usually nonprogressive.8 The symptoms of subacute HP typically develop over weeks to months and include dyspnea, fatigue, and cough. Subacute HP is usually progressive.8
The symptoms of chronic HP are typically insidious and slowly progressive in nature, often culminating in pulmonary fibrosis. This type of presentation is typical with bird antigen exposure. Patients with chronic HP, particularly male smokers, may experience acute exacerbations.8
Serum assays for precipitating antibodies (ie, precipitins) are available and support the diagnosis of HP when positive. However, results are positive in many patients who have been exposed to these antigens but do not have a compatible clinical syndrome of HP. Conversely, a negative result does not rule out HP because the particular antigen causing disease may not have been included in the assay.7 Chest radiography in patients with HP can demonstrate variable findings. In acute and subacute HP, the chest radiograph can be normal but may also show nonspecific findings such as ground glass opacities and fine nodular opacities. The findings of chronic HP are more specific and consist of upper lobe–predominant fibrotic changes such as reticular opacities and honeycombing.7 The typical findings of HP on HRCT imaging include a combination of reticular, ground glass, and centrilobular nodular opacities with fibrotic changes such as interlobular septal thickening, volume loss, traction bronchiectasis, and honeycombing. The fibrotic changes tend to spare the lower lobes, a distinguishing feature between chronic HP and UIP or NSIP.8 The characteristic finding of HP on bronchoscopy with BAL is an increase in total cell count with a dramatic elevation in the percentage of T lymphocytes (>20%, often >50%; normal <5%).7,8
Avoiding exposure to offending antigens is the cornerstone of therapy for HP. Glucocorticoids may alleviate acute symptoms in cases that do not resolve after eliminating antigen exposure, but do not appear to affect long- term outcomes.7
What are the common types of hypersensitivity pneumonitis?
Exposure to moldy hay and grain. Pet parakeets, parrots, or canaries.
Farmer’s lung. Bird fancier’s lung.
961
What antigens are principally responsible for famer’s lung? What antigens are principally responsible for bird fancier’s lung? What are the other types of hypersensitivity pneumonitis?
Farmer’s lung is most commonly associated with antigens from the thermophilic actinomycete species and fungi (eg, Aspergillus species). Farmer’s lung is the prototype of acute HP.8 The antigens associated with bird fancier’s lung come from a mixture of high- and low-molecular weight proteins in avian droppings and dried serum, and on feathers. Bird fancier’s lung is the prototype of chronic HP (but can also cause acute HP).8 Many other types of HP have been described, some of which include woodworker’s lung (from Alternaria species), hot tub lung (from Mycobacterium avium-intracellulare), and chemical worker’s lung (eg, from diisocyanates).
962
Interstitial Lung Disease Related to Pneumoconiosis
What is pneumoconiosis?
Pneumoconiosis describes a disease that occurs as a result of the lung reaction to various inhaled inorganic particles and dust, usually occurring in an occupational setting.9
What are the most common pneumoconioses?
Mining, quarrying, drilling, tunneling, and sandblasting. “Black lung.” Builders, plumbers, electricians, and shipyard workers.
When does silicosis typically develop after exposure?
When does coal worker’s pneumoconiosis typically develop after exposure? What are the findings of silicosis and coal worker’s pneumoconiosis on high-resolution CT imaging? Which granulomatous infectious disease is associated with both silicosis and coal worker’s pneumoconiosis? What is the spectrum of manifestations of asbestos-related lung disease? What are the other pneumoconioses?
Silicosis.
Coal worker’s pneumoconiosis. Asbestosis.
There are 3 main forms of silicosis: acute, chronic, and accelerated. Acute silicosis (ie, acute silicoproteinosis) is rare but can occur within weeks to a few years after initial high-level exposure. It is typically progressive with a poor prognosis. Chronic silicosis (ie, classic silicosis), the most common form, develops insidiously after 10 or more years of low-level exposure. The 2 subtypes of chronic silicosis are simple silicosis and progressive massive fibrosis (PMF); the clinical and imaging features of PMF are more severe. Accelerated silicosis develops 5 to 10 years after initial high- level exposure; clinical and imaging features are similar to chronic silicosis (usually PMF subtype).10 Coal worker’s pneumoconiosis usually presents >10 years after initial exposure and, like chronic silicosis, is characterized by simple and PMF subtypes.11
The HRCT imaging features of silicosis and coal worker’s pneumoconiosis are similar. The most common findings are nodular opacities that are well defined, uniform in shape, and located predominantly in the upper lung zones. The nodules are smaller (<1 cm) in milder forms (ie, simple) and larger (≥1 cm) in more severe forms (ie, PMF).11
Pulmonary and extrapulmonary tuberculosis occur more frequently in patients with silicosis and coal worker’s pneumoconiosis, and the risk increases with severity of lung disease. Cavitation on chest imaging is the strongest indicator of coexistent tuberculosis.9
Asbestos inhalation can cause benign respiratory conditions (eg, pleural plaques and thickening and pleural effusions), interstitial lung disease (ie, asbestosis), and malignant disease (eg, mesothelioma and bronchogenic carcinoma). Asbestosis typically develops 20 to 30 years after exposure with a severity that is proportional to the magnitude and duration of exposure.12 Less common pneumoconioses include berylliosis, hard-metal pneumoconiosis (eg, cobalt), talcosis, and siderosis.9
963
Interstitial Lung Disease Related to Other Exposures
What are the other types of exposure-related interstitial lung disease?
Smokers are at risk for these 3 types of ILD. High risk for right-sided Infective endocarditis.
What are the clinical characteristics of respiratory bronchiolitis– associated interstitial lung disease? What are the clinical characteristics of desquamative interstitial pneumonia? What are the clinical characteristics of pulmonary Langerhans cell histiocytosis? What is the mechanism of interstitial lung disease related to intravenous drug use?
Respiratory bronchiolitis–associated interstitial lung disease (RB-ILD), desquamative interstitial pneumonia (DIP), and pulmonary Langerhans cell histiocytosis (PLCH). Intravenous drug use.
RB-ILD typically affects active smokers between 30 and 40 years of age with ≥30-pack-year history, with a slight predominance in men. Dyspnea and mild cough are the most common symptoms. Inspiratory rales are present in about one-half of cases, whereas digital clubbing is rare. Smoking cessation is the cornerstone of management. The disease usually remains stable or improves, and long-term prognosis is good.13
DIP most commonly affects smokers (90% of cases) in the fourth and fifth decades of life, with a 2:1 predilection for men. Dyspnea and cough are the most common symptoms. Inspiratory rales are present in most, and digital clubbing occurs in about one-half of cases. Smoking cessation is the cornerstone of management, but glucocorticoids may be beneficial in some cases. Prognosis is mixed, with some patients experiencing complete recovery and others progressing despite treatment.13 PLCH affects middle-aged smokers with a slight female predominance. Dyspnea and cough are the most common symptoms. In some cases, there may be involvement of other organ systems (eg, bone, skin, lymph nodes). Smoking cessation is the cornerstone of management and, when instituted early in the course of disease, is usually associated with good prognosis.14 Talc, an agent used to maintain the integrity of medications in tablet form, can become lodged within the lung and cause a foreign-body granulomatous reaction when patients crush and intravenously inject prescription medications.15
964
Interstitial Lung Disease Associated with Systemic Disease
What systemic diseases are associated with interstitial lung disease?
A noninfectious multisystem granulomatous disease that often affects the lungs, producing hilar and mediastinal lymphadenopathy with or without upper lobe–predominant parenchymal changes such as bronchial wall thickening, nodules along the bronchovascular bundles, ground glass opacities, cysts, and fibrosis. Thickened and hardened skin. Associated with antibodies against cyclic- citrullinated peptide. A 48-year-old woman with proximal muscle pain and weakness, elevated serum creatine kinase (CK), heliotrope rash, and “mechanic’s hands” (Figure 47- 4).
Sarcoidosis.
Scleroderma (ie, systemic sclerosis [SSc]). Rheumatoid arthritis (RA). Dermatomyositis (DM).
FIGURE 47-4 Drying and cracking of the skin over the lateral and palmar surfaces of the
A 45-year-old woman with proximal muscle weakness, elevated serum CK, and no skin findings or muscle pain. Among the connective tissue diseases, this is associated with the lowest relative frequency of ILD. A middle-aged woman with dry mouth, dry eyes, and parotitis is found to have serum anti-SSA (ie, anti-Ro) antibodies. A 52-year-old woman with Raynaud’s phenomenon, sclerodactyly, polyarticular inflammatory arthritis, and myositis. Hemoptysis, glomerulonephritis, palpable purpura, and a foot drop.
What are the stages of pulmonary involvement of sarcoidosis?
fingers, known as “mechanic’s hands,” is seen frequently in the antisynthetase syndrome that occurs in some patients with dermatomyositis or polymyositis. (From Koopman WJ, Moreland LW. Arthritis and Allied Conditions A Textbook of Rheumatology. 15th ed. Philadelphia, PA: Lippincott Williams & Wilkins; 2005.)
Polymyositis (PM).
Systemic lupus erythematosus (SLE).16
Sjögren’s syndrome.
Mixed connective tissue disease (MCTD).
Vasculitis.
There are 4 stages of pulmonary involvement of sarcoidosis, according to radiographic evaluation: stage 1, bilateral hilar and mediastinal lymphadenopathy without parenchymal disease; stage 2, bilateral hilar lymphadenopathy with upper lobe–predominant parenchymal disease with distinctive features (eg, nodules along bronchovascular bundles); stage 3, parenchymal disease alone (shrinking or absent hilar lymphadenopathy); and stage 4, fibrosis, often upper lobe predominant, usually accompanied by volume loss, traction bronchiectasis, cysts, and diaphragmatic tenting (around 10% of patients progress to this stage). The ILD of
965
How common is interstitial lung disease in patients with systemic sclerosis?
What are the characteristics of interstitial lung disease associated with rheumatoid arthritis?
How common is interstitial lung disease in patients with dermatomyositis or polymyositis?
What populations with systemic lupus erythematosus are at higher risk for interstitial lung disease? What are the main risk factors for pulmonary involvement related to Sjögren’s syndrome? How common is interstitial lung disease in patients with mixed connective tissue disease? Which vasculitides are associated with interstitial lung disease?
sarcoidosis is associated with a distinctive pattern on HRCT imaging.2,17 A significant proportion of patients with SSc develop ILD. It is more common in patients with diffuse SSc (approximately 55%) compared with those with limited SSc (approximately 35%). The ILD of SSc most commonly manifests as NSIP or UIP.16 ILD is the most common pulmonary manifestation of RA, occurring more frequently in men (2:1), with onset most often in the fifth to sixth decades of life. The occurrence of ILD decreases survival in patients with RA from around 10 years to 2.5 years from the time of diagnosis. The ILD of RA most commonly manifests as UIP or NSIP.18 ILD occurs in up to half of patients with DM or PM and is predicted by the presence of autoantibodies to aminoacyl-tRNA synthetase enzymes (eg, anti-histidyl-tRNA synthetase [anti-Jo-1]), which is associated with the antisynthetase syndrome. The ILD of DM and PM most commonly manifests as NSIP, UIP, or COP.16 Among patients with SLE, ILD is more common in those who are older, male, and have late-onset disease. The ILD of SLE most commonly manifests as AIP. Given the higher prevalence of SLE in women, more women will be affected by ILD despite the relative higher frequency in men with SLE.19 Risk factors for pulmonary involvement in Sjögren’s syndrome include male sex, smoking, late onset, and long evolution of disease. The ILD of Sjögren’s syndrome most commonly manifests as LIP, NSIP, UIP, or COP.20 ILD complicates MCTD in about one-half of patients (around 20% of these cases result in severe fibrosis). The occurrence of ILD in patients with MCTD results in Increased mortality. The ILD of MCTD most commonly manifests as NSIP.16,21 ILD is associated with antineutrophil cytoplasmic antibody–associated vasculitis, particularly microscopic polyangiitis—it is frequently present at the time of diagnosis. The ILD of vasculitis most commonly manifests as UIP.22
966 Case Summary A 58-year-old man presents with chronic and progressive dyspnea and weight loss and is found to have hypoxemia, abnormalities of th
hands, a loud P2, a diastolic murmur, fine end-inspiratory rales, ande abnormal cross-sectional imaging of the chest.
What is the most likely diagnosis in this patient? Interstitial lung disease associated with scleroderma.
967
Bonus Questions
What is scleroderma? Scleroderma refers to a group of related conditions that result in cutaneous thickening and sclerosis (ie, hardening of the skin) and digital cutaneous ulcers. Depending on the subtype of scleroderma, there may be a variety of other manifestations, including internal organ involvement (eg, esophageal dysmotility). What are the 2 main types of Scleroderma can be localized (eg, morphea) or systemic (SSc), the latter of which is frequently associated with internal organ involvement. be scleroderma? There are 2 main subtypes of SSc: limited cutaneous SSc (around 60% of cases) and diffuse cutaneous SSc (around 35% of cases). There can considerable overlap between these subtypes. 23 What are the main features of limited Limited cutaneous SSc involves the skin but is typically limited to the hands, forearms, and face. Serum antinuclear antibodies and cutaneous systemic sclerosis? anticentromere antibodies are present in most cases. 23 What are the main features of diffuse Diffuse cutaneous SSc is more rapidly progressive than the limited form. Cutaneous involvement may extend to the proximal arms, trunk, and cutaneous systemic sclerosis? face. Other than antinuclear antibody, anti-Scl-70 is the most frequent antibody in diffuse cutaneous SSc. Early and severe internal organ involvement (eg, Gastrointestinal tract, kidneys, lungs, heart) is typical.23 What is the nature of lung involvementic Limited cutaneous SSc is more closely associated with pulmonary hypertension, whereas diffuse cutaneous SSc is more closely associated with sclerosis? in limited and diffuse cutaneous system ILD. However, there is considerable overlap, and both manifestations may be present in some cases. 23,24 What findings are demonstrated in the The photograph of the pat ent’s hands in this case (see Figure 47-1) shows sclerodactyly, which is characterized by skin thicken ng and image of the patient’s hands in this case? hardening, giving the skinia shiny appearance and leading to curling of the fingers with loss of mobility. There is also evidenceiof digital skin What are the major findings on high-this HRCT imaging in this case (see Figure 47-2A and B) shows lowerflobe– predominanttsubpleural reticular opacities and honeycombing pitting and poorly healing ulcerations. case? resolution CT imaging of the chest in (arrows), traction bronchiectasis (arrowheads), and the absence o ground glass opaci ies. The imaging pattern in this case isform The pattern on HRCT imaging in this case is that of UIP, which is a common presentation of ILD associated with scleroderma. characteristic of which idiopathic of interstitial lung disease? Based on the physical findings, whichely The loud pulmonic component of the second heart sound (P2) is suggestive of pulmonary hypertension, which can be associated with SSc. What is the significance of the diastolic Pulmonic valve insufficiency can occur as a result of pulmonary hypertension. In this setting, it is referred to as a Graham Steell murmur. additional pulmonary condition is lik to be present in this case? murmur in this case? How common is interstitial lung disease Interstitial changes are found in the vast majority of patients with SSc, but clinically significant pulmonary fibrosis is seen in about one-quarter in patients with systemic sclerosis? of cases. 24 What is the prognosis of interstitial lung Treatment for scleroderma-associated ILD, typically with cyclophosphamide, is only of modest benefit. There is variability in prognosis, but sclerosis?24 pattern (median survival of 3 years). disease associated with systemic overall, it is poor. Patients with an NSIP pattern tend to have better outcomes (median survival of 15 years) compared with those with a UIP
968 Key Points
parenchyma that produce characteristic clinical, physiologic, ILD describes a group of heterogeneous diseases of the lung imaging, and histologic manifestations.
The most common clinical manifestations of ILD include dyspnea, dry cough, bibasilar fine end-inspiratory rales, and digital clubbing.
ILD can be acute and rapidly progressive or insidious in onset, HRCT imaging is useful in the evaluation of ILD and can identify ILD usually manifests as a restrictive lung disease with impaired depending on the subtype. patterns of findings associated with particular subtypes of ILD. diffusion capacity on pulmonary function testing. Bronchoscopy or surgical biopsy (including VATS) can provide a
histologic diagnosis and can also evaluate for alternative etiologies (eg, infection). ILD can be idiopathic or secondary to other conditions.
Idiopathic ILD is not associated with any identifiable secondary The various idiopathic forms of ILD are associated with distinctive The 2 most common morphologic patterns of ILD are UIP and Secondary ILD can occur as a result of exposure or systemic condition. imaging and histologic patterns. NSIP. disease. The causes of exposure-related ILD can be separated into the
following subcategories: iatrogenic, HP, pneumoconiosis, and The secondary forms of ILD, particularly systemic diseases, often other. present with the morphologic patterns of the idiopathic forms of ILD.
Clinical manifestations and prognosis of ILD are highly variable d and depend on the specific subtype, ranging from self-limited an reversible to rapidly progressive and fatal.
969
References 1. Longo DL, Fauci AS, Kasper DL, Hauser SL, Jameson JL, Loscalzo J, eds. Harrison’s Principles of Internal Medicine. 18th ed. New York, NY: McGraw-Hill; 2012.
-
Wallis A, Spinks K. The diagnosis and management of interstitial lung diseases. BMJ. 2015;350:h2072. Raghu G, Collard HR, Egan JJ, et al. An official ATS/ERS/JRS/ALAT sta eme
-
idiopathic pulmonary fibrosis: evidence-based guidelines for diagnosistand nt: management. Am J Respir Crit Care Med. 2011;183(6):788-824. Raghu G, Rochwerg B, Zhang Y, et al. An official ATS/ERS/JRS/ALAT clinical practic
-
guideline: treatment of idiopathic pulmonary fibrosis. An update.of the 2011 clinical e 5. radiologist should know about idiopathic interstitial pneumonias. Radiographics. practice guideline. Am J Respir Crit Care Med. 2015;192(2):e3-e19 Mueller-Mang C, Grosse C, Schmid K, Stiebellehner L, Bankier AA. What every 2007;27(3):595-615.
-
Williams JP, Johnston CJ, Finkelstein JN. Treatment for radiation-induced pulmonary late effects: spoiled for choice or looking in the wrong direction? Curr Drug Targets.
2010;11(11):1386-1394. 7. Spagnolo P, Rossi G, Cavazza A, et al. Hypersensitivity pneumonitis: a comprehensive review. J Investig Allergol Clin Immunol. 2015;25(4):237-250; quiz follow 50.
-
Selman M, Pardo A, King TE Jr. Hypersensitivity pneumonitis: insights in diagnosis and pathobiology. Am J Respir Crit Care Med. 2012;186(4):314-324.
-
Chong S, Lee KS, ChungfMJ, Han J, Kwon OJ, Kim TS. Pneumoconiosis: comparison of 10. Leung CC, Yu IT, Chen W. Silicosis. Lancet. 2012;379(9830):2008-2018. imaging and pathologic indings. Radiographics. 2006;26(1):59-77.
-
Laney AS, Weissman DN. Respiratory diseases caused by coal mine dust. J Occup Environ Med. 2014;56(suppl 10):S18-S22.
-
Currie GP, Watt SJ, Maskell NA. An overview of how asbestos exposure affects the lung. BMJ. 2009;339:b3209. . Attili AK, Kazerooni EA, Gross BH, Flaherty KR, Myers JL, Martinez FJ.
13related interstitial lung disease: radiologic-clinical-pathologic correlation.Smoking- Radiographics. 2008;28(5):1383-1396; discussion 96-98. . Elia D, Torre O, Cassandro R, Caminati A, Harari S. Pulmonary Langerhans ce l
14histiocytosis: a comprehensive.analysis of 40 patients and literature review. EurlJ 15hypertension.resulting from intravenous injection of talc-containing drugs intended for Intern Med. 2015;26(5):351-356 . Roberts WC Pulmonary talc granulomas, pulmonary fibrosis, and pulmonary oral use. Proc (Bayl Univ Med Cent). 2002;15(3):260-261.
- Castelino FV, Varga J. Interstitial lung diseasetin connective tissue diseases: evolving concepts of pathogenesis and management. Ar hritis Res Ther. 2010;12(4):213.
-
Iannuzzi MC, Rybicki BA, Teirstein AS. Sarcoidosis. N Engl J Med. 2007;357(21):2153- 2165.
-
Shaw M, Collins BF, Ho LA, Raghu G. Rheumatoid arthritis-associated lung disease. Eur Respir Rev. 2015;24(135):1-16.
-
Cheema GS, Quismorio FP Jr. Interstitial lung disease in systemic lupus erythematosus. Curr Opin Pulm Med. 2000;6(5):424-429.
-
Flament T, Bigot A, Chaigne B, Henique H, Diot E, Marchand-Adam S. Pulmonary
970
manifestations of Sjogren’s syndrome. Eur Respir Rev. 2016;25(140):110-123. 21lung disease in mixed;connective tissue disease: a nationwide, cross-sectional study.al . Gunnarsson R, Aalokken TM, Molberg O, et al. Prevalence and severity of interstiti Ann Rheum Dis. 2012 71(12):1966-1972. . Katsumata Y, Kawaguchi Y, Yamanaka H. Intersti ial lung disease with ANCA- 22associated vasculitis. Clin Med Insights Circ RespirtPulm Med. 2015;9(suppl 1):51-56. 23. Hinchcliff M,iVarga J. Systemic sclerosis/scleroderma: a treatable multisystem disease. 24. Schoenfeld SR, Castelino FV. Interstitial lung disease in scleroderma. Rheum Dis Clin Am Fam Phys cian. 2008;78(8):961-968. North Am. 2015;41(2):237-248.
971
CHAPTER 48
972
Pleural Effusion
973 Case: A 62-year-old man with pleuritic chest pain A 62-year-old Filipino man who underwent coronary artery bypass department with increasing shortness of breath. The patient had done progressive dyspnea on exertion. He also describes sharp left-sidedby Other than coronary artery disease, the patient has no known medical grafting (CABG) surgery 5 months ago presents to the emergency well in the months after surgery until 1 week before presentation when he developed fever, chills, and dry cough. This was followed chest discomfort that worsens with deep breathing and coughing. problems. He takes aspirin and metoprolol but no other medications. The patient was born in the Philippines and lived there before
emigrating to the United States 2 years ago. He has never smoked cigarettes. Temperature is 38.3°C, and respiratory rate is 26 breaths per minut
The patient appears acutely ill and is diaphoretic. There is Decreasede. dullness to percussion and absent tactile fremitus. There are tubular rales. Chest radiographs are shown in Figure 48-1A (current) and inspiratory excursion of the left posterior chest wall along with breath sounds just above the region of dullness but no inspiratory Figure 48-1B (before surgery). A thoracentesis is performed, and pleural fluid analysis shows a
leukocyte count of 2600 cells/µL with a lymphocyte fraction of 95%. concentration is 8.1 g/dL). Pleural fluid Gram stain and Ziehl-Neelsen pending. Cytologic examination of the fluid isinegative for malignant Pleural fluid protein concentration is 5.8 g/dL (serum total protein stain for acid-fast bacilli do not show any organisms. Culture is cells. Pleural fluid adenosine deaminase level s 6.3 U/L (reference range 0 to 9.4 U/L).
FIGURE 48-1
What is the most likely cause of the pleural effusion in this patient?
What is a pleural effusion? A pleural effusion is the abnormal accumulation of fluid in the pleural cavity, usually as a result of Increased pleural fluid production, Decreased pleural fluid absorption, or both (Figure 48-2).1
974
What are the mechanisms of pleural fluid accumulation? How common are pleural effusions? What are the symptoms of pleural effusions? What physical findings are associated with pleural effusions? What is the role of chest radiography in the evaluation of pleural effusions? What is the definition of a large pleural effusion? What radiographic clue can be used to determine if complete hemithorax opacification is related to atelectasis or a space- occupying process? What is the role of computed tomography (CT) imaging in the evaluation of pleural effusions? What is a loculated pleural effusion?
FIGURE 48-2 Pleural effusion describes the abnormal accumulation of fluid in the pleural space. (From Pellico LH. Focus on Adult Health: Medical-Surgical Nursing. Philadelphia, PA: Wolters Kluwer Health; 2013.)
The main mechanisms of pleural effusion accumulation are Increased pulmonary capillary and interstitial hydrostatic pressure, Decreased capillary oncotic pressure, Increased pleural membrane permeability, Decreased intrapleural pressure, and obstruction to lymphatic flow.2 Pleural effusions are common, with an incidence of approximately 400 cases per 100,000 persons per year in the industrialized world.1 Symptoms of pleural effusions depend on the size and rate of accumulation but may include dyspnea, nonproductive cough, and chest pain (usually pleuritic).1,2 Physical findings of pleural effusions may include Decreased inspiratory expansion of the chest wall on the affected side, dullness to percussion, Decreased tactile fremitus, and reduced breath sounds over the effusion. Chest radiography is sensitive for the presence of a pleural effusion, which appears as a meniscus on the lateral view when the volume is >50 mL. A meniscus becomes visible on the posteroanterior view when the volume is >200 mL; obscuration of the hemidiaphragm occurs when the volume is >500 mL.3 A pleural effusion is considered large when it occupies more than one-quarter of the hemithorax.4 On a chest radiograph, tracheal deviation toward the opacified hemithorax suggests volume loss over that area (ie, collapsed lung); tracheal deviation away from the opacified hemithorax suggests a volume- occupying process (eg, pleural effusion).
Contrast enhanced CT imaging provides more information than conventional radiography. In addition to the pleural effusion(s), it may identify findings such as pleural thickening and nodularity, and parenchymal lesions not visible with chest radiography.1
A loculated pleural effusion does not flow freely within the pleural cavity and can be associated with certain types of effusions. A chest radiograph taken in the lateral decubitus position can identify whether the fluid is flowing freely or not (Figure 48-3). Ultrasonography and CT imaging have largely supplanted this technique and are now increasingly used to identify loculation.
FIGURE 48-3 A, Chest radiograph showing a moderate right-sided pleural effusion. B, Right lateral decubitus
What is an empyema?
What are the 2 general types of pleural effusions?
What procedure must be performed to determine whether pleural fluid is transudative or exudative? What validated criteria can
radiograph confirming that the right pleural effusion (arrows) is free flowing and not loculated. (From Smith WL, Farrell TA. Radiology 101: The Basics and Fundamentals of Imaging. 4th ed. Philadelphia, PA: Lippincott Williams & Wilkins; 2014.)
An empyema is defined by the presence of pus in the pleural space. The fluid is thick, viscous, and purulent in appearance.5 Pleural effusions can be transudative or exudative.
Thoracentesis allows for the evaluation of pleural fluid, including the determination of its transudative or exudative nature. Therapeutic thoracentesis can also be used to relieve symptoms in patients with refractory or recurrent pleural effusions. Pleural fluid is considered exudative when at least 1 of the following 3 conditions are met: (1) ratio of pleural
975
help determine whether transudative or exudative? pleural fluid is What are the operating characteristics of Light’s criteria?
fluid protein level to serum proteinllevel >0.5, (2) ratio of pleural fluid lactate dehydrogenase (LDH) level to serum LDH level >0.6, or (3) pleura fluid LDH level >two-thirds the upper limit of normal for serum LDH level. These conditions are collectively known as Light’s criteria.6,7 The use of Light’s criteria is 98% sensitive for identifying exudates, but specificity is lower at 83%. This means that 17% of true transudates will be misclassified as exudates. In cases where an effusion is unexpectedly classified as an exudate by Light’s criteria (ie, the clinical picture suggests a transudate), then the difference between albumin levels in serum and pleural fluid should be calculated by subtracting pleural fluid albumin from serum albumin. If the difference is >1.2 g/dL, the effusion is transudative in virtually all cases.6
What basic Routine pleural fluid evaluation includes the gross appearance of the fluid (color, turbidity, viscosity); total and fluid differential cell counts; protein, LDH, and glucose levels; and smear and culture. Other tests are available and should be characteristics used in the appropriate settings (eg, pH if empyema is suspected, cytology if malignancy is suspected, adenosine can be helpful deaminase concentration if tuberculosis [TB] is suspected). in the evaluation of pleural effusions? What is the The appearance of pleural fluid can help narrow the differential diagnosis. For example, bloody effusions are most significance commonly associated with hemothorax, malignancy, pulmonary embolism (PE), trauma, and pneumonia. Tenacious and of the gross cloudy fluid suggests infection. Fluid that is milky in appearance suggests chylothorax.6 appearance of pleural fluid? What is the The predominant cell type in pleural fluid can help narrow the differential diagnosis. A predominance of neutrophils significance (>50%) suggests an acute process, such as pneumonia, PE, pancreatitis, or intra-abdominal abscess, whereas a of the predominance of mononuclear cells suggests a chronic process. A predominance of lymphocytes is suggestive of differential malignancy, tuberculous pleuritis, postcardiac injury syndrome (PCIS), or post-CABG (some of these conditions may be cell count of associated with an initial predominance of neutrophils). A predominance of eosinophils can be suggestive of drug- pleural fluid? induced effusions, hemothorax, asbestos exposure, and eosinophilic granulomatosis with polyangiitis (EGPA, or Churg- Strauss syndrome).6 What is the Low pleural fluid glucose concentration (<60 mg/dL) is suggestive of parapneumonic effusion or malignancy. Other less significance common causes include hemothorax, tuberculous pleuritis, and rheumatoid pleuritis.6 of the glucose concentration of pleural fluid? What is the Gram stain and culture of pleural fluid should be included in any infectious workup. Culture bottles should be inoculated significance at the bedside. Special smears and cultures may be indicated based on other clinical data (eg, acid-fast smear if TB is of Gram stain suspected).6 and culture of pleural fluid? What is the Cytologic examination of pleural fluid can establish a diagnosis of malignancy. Yield is variable and dependent on the significance type of cancer. For example, the sensitivity of cytologic examination is as high as 70% for metastatic adenocarcinoma but of cytologic as low as 10% for mesothelioma.6 examination of pleural fluid?
976
Transudative Pleural Effusions
What are the 3 main mechanisms of transudative pleural effusions?
What is the role of hydrostatic pressure in the development of a pleural effusion? What is the role of intrapleural pressure in the development of a pleural effusion? What is the role of capillary oncotic pressure in the development of a pleural effusion? What is the role of the diaphragm in the development of a pleural effusion?
Transudative pleural effusions can occur as a result of changes in hydrostatic pressure, changes in oncotic pressure, or diaphragmatic defects.
An increase in capillary hydrostatic pressure, which opposes capillary oncotic pressure and intrapleural pressure, will lead to efflux of fluid from the capillaries to the pleural space.
Intrapleural pressure opposes both capillary and interstitial hydrostatic pressure, acting to prevent influx of fluid into the pleural space. A decrease in intrapleural pressure will lead to a relative increase in capillary and interstitial hydrostatic pressure and an efflux of fluid from the capillaries and interstitium to the pleural space. Capillary oncotic pressure opposes capillary hydrostatic pressure, acting to maintain fluid within the capillaries. A decrease in capillary oncotic pressure will lead to efflux of fluid from the capillaries to the interstitial and pleural spaces. Small defects in the diaphragm can allow peritoneal fluid to move into the pleural space.
977
Transudative Pleural Effusions Related to Hydrostatic Pressure
What are the causes of transudative pleural effusions related to hydrostatic pressure?
More than 80% of effusions related to this condition are bilateral. External cardiac restraint. Elevated jugular venous pressure, dilated chest wall veins, and Horner’s syndrome. These entities result in Decreased intrapleural pressure, creating a relative increase in capillary and interstitial hydrostatic pressure, resulting in the movement of fluid into the pleural space. Drainage of pleural fluid related to this entity will invariably result in pneumothorax.
What is the management of pleural effusions related to congestive heart failure? What are the characteristics of pleural effusions related to constrictive pericarditis? What types of pleural effusions can develop in patients with superior vena cava syndrome? Why are pleural effusions associated with atelectasis? When does pleural fluid usually accumulate after pneumonectomy?
What is trapped lung?
Congestive heart failure (CHF). All causes of volume overload, including renal failure and intravenous fluid administration, are described by this term.6 Constrictive pericarditis. Superior vena cava (SVC) syndrome.
Atelectasis and pneumonectomy.
Trapped lung.
Diuretic therapy is the cornerstone of treating pleural effusions related to CHF. Within 48 hours of initiating diuresis, most effusions decrease or resolve. Pleural effusions related to CHF may be exudative in patients with a history of CABG or may become exudative after aggressive diuretic therapy.6,8 Pleural effusions occur in about one-half of patients with constrictive pericarditis. Most are bilateral, but unilateral effusions affecting either the right or the left side occur.9 In addition to transudative effusions, exudative effusions (including chylous effusions) have been described in adult patients with SVC syndrome. The mechanisms are likely multifactorial, including lymphatic obstruction.10 A pleural effusion of any cause can result in passive atelectasis, which can often be appreciated on physical examination as signs of consolidation (eg, egophony) just above the level of the effusion. The rate of pleural fluid accumulation after pneumonectomy is variable. Typically, half of the pleural space is filled with fluid within the first 4 to 5 days after pneumonectomy; fluid gradually increases until the space is filled over a period of weeks to months. If more rapid filling is detected, a surgical complication resulting in hemothorax or chylothorax should be investigated. If filling is tardy or stops, a bronchial anastomotic leak should be suspected.11 Trapped lung describes the inability of lung to re-expand as a result of fibrous visceral pleural thickening that can develop from a chronic inflammatory pleural process. As pleural fluid surrounding trapped lung is removed via thoracentesis, intrapleural pressures become increasingly negative because lung cannot re-expand to fill the space. This creates a vacuum within the pleural space that pulls in air from around the catheter (known as pneumothorax ex vacuo).12
978
Transudative Pleural Effusions Related to Oncotic Pressure
What are the causes of transudative pleural effusions related to oncotic pressure?
A reduction in albumin synthesis. A 38-year-old man with human immunodeficiency virus (HIV) infection presents with anasarca and foamy urine. Albumin loss from the Gastrointestinal tract. An Endocrinopathy.
Is hypoalbuminemia the only mechanism of pleural effusion formation related to cirrhosis? What pulmonary condition should be suspected in patients with nephrotic syndrome who present with sudden-onset dyspnea and chest pain? What stool study can be helpful in the diagnosis of protein-losing enteropathy? What is the treatment for pleural effusions related to myxedema?
Cirrhosis. Nephrotic syndrome.
Protein-losing enteropathy (PLE). Myxedema.
In addition to hypoalbuminemia, cirrhosis can also cause a pleural effusion via movement of ascitic fluid into the pleural space through diaphragmatic defects (ie, hepatic hydrothorax).
Pulmonary embolism is a known complication of nephrotic syndrome (particularly membranous glomerulonephropathy) and should be suspected when these patients present with sudden-onset pulmonary symptoms. Pleural effusions associated with nephrotic syndrome tend to develop over time and are often associated with peripheral edema and ascites. The effusions are bilateral and usually responsive to diuretic agents in addition to management of the underlying kidney disease.13 Increased fecal α-1 antitrypsin clearance is suggestive of PLE. The clinical characteristics of pleural effusions associated with PLE are similar to those of nephrotic syndrome. Most pleural effusions related to myxedema are not clinically relevant or symptomatic, and respond to thyroid replacement therapy. Pleural effusions related to myxedema can be either transudative or exudative.14
979
Transudative Pleural Effusions Related to Diaphragmatic Defects
What are the causes of transudative pleural effusions related to diaphragmatic defects?
There is usually obvious ascites associated with this condition, but sometimes it is inconspicuous. An iatrogenic cause. A middle-aged man presents with a new pleural effusion 1 month after percutaneous nephrolithotomy.
Hepatic hydrothorax.
Peritoneal dialysis. Urinothorax.
What are the Hepatic hydrothorax occurs in up to 10% of patients with cirrhosis. Most cases occur on the right characteristics of (approximately 85%) and are usually >500 mL in volume. As with ascitic fluid, spontaneous infection of the hepatic hydrothorax? pleural fluid, called spontaneous bacterial empyema, can occur. It is defined as a nonparapneumonic pleural effusion with a polymorphonuclear (PMN) cell count >500 cells/µL, or a PMN cell count >250 cells/µL with positive culture.15 What are the Pleural effusions occur in around 2% of peritoneal dialysis patients. It typically occurs on the right but can occur characteristics of pleural on the left or on both sides in some cases. Sometimes referred to as “sweet hydrothorax,” the fluid has effusions related to characteristically high glucose content, which is a clue to the diagnosis.16 peritoneal dialysis? What pleural fluid A fluid-to-serum creatinine ratio >1.0 is diagnostic of urinothorax.17 characteristic is pathognomonic for urinothorax?
980
Exudative Pleural Effusions
What are the 2 main mechanisms of exudative pleural effusions? What are the 2 general types of exudative pleural effusions?
Why is it important to distinguish infection from other causes of exudative pleural effusions?
Exudative pleural effusions can occur as a result of Increased capillary permeability or lymphatic obstruction. Exudative pleural effusions can be infectious or noninfectious.
It is important to recognize infectious pleural effusions because urgent drainage may be necessary in some cases.
981
Infectious Pleural Effusions
What are the infectious causes of exudative pleural effusions?
Fever, dyspnea, purulent cough, and consolidation with an associated pleural effusion on chest imaging. One of the most common extrapulmonary manifestations of this granulomatous disease. A 60-year-old woman with immune thrombocytopenic purpura who recently underwent splenectomy is admitted with fever and abdominal pain and is found to have an exudative pleural effusion with a neutrophil cell fraction of 90% without evidence of an associated parenchymal consolidation.
What features of pleural effusions associated with pneumonia (ie, parapneumonic effusion) are important for deciding whether the fluid should be sampled? When a parapneumonic effusion is diagnosed and drained with thoracentesis, what urgent clinical determination must be made next?
Is mycobacterial culture from sputum samples helpful in the diagnosis of tuberculous pleuritis when there is no discernable parenchymal involvement? What are the characteristics of the pleural effusion related to a subphrenic abscess?
Pneumonia.
Tuberculous pleuritis. Subphrenic abscess.
It is said that “the sun should never set on a parapneumonic effusion.” Thoracentesis is indicated for all parapneumonic effusions except those that are free flowing and less than 10 mm thick on a lateral decubitus chest radiograph. These effusions are associated with low risk of poor outcomes and do not require drainage.5,18 It must be determined if a parapneumonic effusion is complicated, as such effusions are likely to require an additional, more invasive procedure (eg, tube thoracostomy) for resolution and prevention of trapped lung. A pleural effusion is considered to be complicated if any of the following are present: (1) frank pus in the pleural space, (2) positive Gram stain or culture of the fluid, (3) fluid pH <7.2, (4) fluid glucose <60 mg/dL, or (5) the fluid occupies more than one-half the hemithorax, is loculated, or is associated with thickened parietal pleura on imaging.5 Sputum mycobacterial cultures can be helpful even in patients without obvious parenchymal involvement on chest imaging, with a yield of up to 55%. Pleural fluid studies are associated with variable sensitivity. Pleural biopsy may ultimately be necessary to make the diagnosis.19 A pleural effusion occurs in most cases of subphrenic abscess. It is typically small to moderate in size, and there is a predominance of neutrophils. The presentation is often dominated by thoracic symptoms, such as pleuritic chest pain; in fact, around one-third of patients with subphrenic abscess do not report any abdominal pain.14
982
Noninfectious Exudative Pleural Effusions
What are the noninfectious causes of exudative pleural effusions?
A 65-year-old man who is an active smoker presents with hemoptysis, weight loss, and a new pleural effusion. Unilateral leg swelling, hemoptysis, and a pleural effusion. A young woman with malar rash, pleuritic chest pain, and a new pleural effusion. A 44-year-old man with heavy alcohol use presents with epigastric abdominal pain radiating to the back and a new pleural effusion. A 24-year-old man presents to the emergency department with dyspnea and chest pain following a motor vehicle accident, during which he hit his chest against the dashboard, and is found to have dullness to percussion of the left hemithorax and Decreased tactile fremitus. Milky appearing pleural fluid. A 56-year-old man who sustained a myocardial infarction 3 weeks ago presents with fever and pleuritic chest pain and is found to have pericarditis and a pleural effusion. A history of coronary artery disease is a prerequisite. Medication review is part of taking a good history. Pleural effusions related to this treatment could be mistaken for disease recurrence. An environmental exposure commonly associated with the development of pleural plaques. Associated with delirium and asterixis on examination. Ascites, a pleural effusion, and a benign ovarian tumor. Yellow fingernails, lymphedema, and a pleural effusion.
What types of malignancies are most often associated with pleural effusions?
What radiographic findings are associated with pulmonary embolism?
Malignancy.
Pulmonary embolism. Systemic lupus erythematosus (SLE).
Acute pancreatitis.
Hemothorax.
Chylothorax. Postcardiac injury syndrome (PCIS).
Post-CABG. Medication. External beam radiation therapy for intrathoracic malignancy. Asbestos.
Uremia. Meigs’ syndrome. Yellow nail syndrome.
In the industrialized world, lung cancer, breast cancer, and lymphoma cause the majority of malignant pleural effusions. Other causative malignancies include metastatic ovarian cancer, sarcoma, and melanoma. Fluid cytology examination may be diagnostic. The presence of a pleural effusion in a patient with lung cancer almost always indicates noncurable disease even in the absence of positive cytology.14 The chest radiograph is typically clear in the setting of PE but may show a wedge-shaped opacity in the periphery of the lung abutting the pleura indicative of infarction (Hampton’s hump) or oligemia distal to the embolus (Westermark sign). Pleural effusions associated with pulmonary embolism are typically small. It is often asserted that pulmonary embolism can cause both transudative and exudative pleural effusions, but the vast majority are exudative.14
983
What additional tests should be Because of the risk of lymphoma in patients with Sjögren’s syndrome and the fact that considered in patients with Sjögren’s lymphoma is a leading cause of malignant pleural effusion, additional tests such as cytology, syndrome who present with a pleural flow cytometry, and pleural biopsy should be considered in these patients. effusion? What pleural fluid study can be useful in Pleural fluid amylase level can be elevated when the effusion is related to either acute cases of acute pancreatitis or esophageal pancreatitis or esophageal rupture (ie, Boerhaave’s syndrome).6 rupture? When hemothorax is suspected based on When the pleural fluid hematocrit level is <1%, the presence of blood is insignificant. A pleural the appearance of the fluid, what fluid hematocrit level above 1% is most often caused by malignancy with pleural involvement, laboratory test can confirm the pulmonary embolism, or trauma. Hemothorax can be confirmed when the pleural fluid diagnosis? hematocrit level is greater than one-half that of blood. Tube thoracostomy should be considered in cases of hemothorax. Although not always accurate, an estimate of the hematocrit of the fluid can be obtained by dividing the red blood cell (RBC) count by 100,000 (eg, RBC count of 1,000,000 =hematocrit of 10).2,14 What pleural fluid study can be useful in The pleural fluid triglyceride level is helpful in diagnosing chylothorax. If the level is the evaluation of chylothorax? >110 mg/dL, there is a high likelihood of chylothorax; if the level is <50 mg/dL, chylothorax is ruled out. For values in between, lipoprotein electrophoresis can be performed.14 What are some causes of postcardiac PCIS can be caused by postmyocardial infarction syndrome (ie, Dressler’s syndrome), cardiac injury syndrome? surgery, blunt chest trauma, pacemaker implantation, and angioplasty.4 Which hemithorax is most often involved The left hemithorax is most often involved in post-CABG pleural effusions.4 in post-CABG pleural effusions? What medications can cause pleural Medications most commonly associated with pleural effusions include amiodarone, effusions? methotrexate, phenytoin, sirolimus, and nitrofurantoin.2 What are the characteristics of radiation- Radiation-induced pleural effusions most commonly occur in association with radiation induced pleural effusions? pneumonitis, affecting the ipsilateral lung. Unlike malignant effusions, radiation-related effusions typically do not increase in size over time, which can help inform the diagnosis when considering cancer recurrence in patients who develop a pleural effusion after radiation therapy.20 What is the prognosis of asbestos-related The vast majority of asbestos-related pleural effusions are benign, but since asbestos is pleural effusions? associated with lung cancer (including mesothelioma), the effusion should be thoroughly evaluated. Most benign asbestos effusions are small and unilateral, and resolve spontaneously after a few months. Recurrences are rare.21 What are the characteristics of pleural Pleural effusions occur in around 3% of uremic patients and are most frequently unilateral effusions related to uremia? (approximately 20% are bilateral). Most patients are symptomatic with fever, chest pain, cough, and dyspnea. The size of the effusion can be variable but is frequently large.14 What is the definitive treatment for the Surgical removal of the ovarian tumor usually results in resolution of the pleural effusion pleural effusion associated with Meigs’ associated with Meigs’ syndrome.22 syndrome? What are the characteristics of pleural The pleural effusions of yellow nail syndrome are bilateral in about one-half of cases and vary effusions related to yellow nail in size from small to massive. There is typically a predominance of lymphocytes. Persistence syndrome? and rapid recurrence of the effusion after thoracentesis is typical. There is no specific treatment for the disease, but pleurodesis can be considered for symptomatic cases.14
984 Case Summary presents with fever, dyspnea, and chest pain and is found to have a ry A 62-year-old Filipino man who recently underwent CABG surge large left pleural effusion.
What is the most likely cause of the pleural effusion in this patient? Postcardiac injury syndrome.
985
Bonus Questions
How does the the The 2 most relevant pieces of history in this case include the patient’s country of origin, which has a high burden of tuberculosis, and recent cardiac surgery, which diagnosis in this differential history shape can lead to PCIS. case? Based on the The pleural fluid protein to serum protein ratio is >0.5, meeting Light’s criteria for an exudative effusion. information provided in this
case, is the pleural transudative or effusion exudative? What is the of the A lymphocyte-predominant exudative pleural effusion narrows the differential diagnosis primarily to malignancy, tuberculous pleuritis, and PCIS. Based on the the fluid in thisf lymphocytic predominance o significance history in this case, tuberculosis and PCIS should already have been under consideration. case? What is the Pleural fluid adenosine deaminase level, which increases in the setting of TB, has excellent operating characteristics. A cutoff value of 40 U/L is associated with
significance of the sensitivity and specificity that exceed 90% for the diagnosis of tuberculous pleuritis. The yield of pleural fluid smear for acid-fast bacilli is poor except in patients fluid adenosine infected with HIV and those with tuberculous empyema. Pleural fluid culture has a low sensitivity and is limited by lengthy delays in obtaining results. Pleural deaminase test in fluid nucleiclacid amplificationitests are also associated with poor sensitivity. Sputum cultures are positive in up to one-half of cases, even in the absence oftobvious this case? parenchyma disease. Pleural b opsy may ultimately be necessary to definitively rule out tuberculous pleuritis in cases where suspicion remains high despi e negative pleural fluid studies. 6,23 How common isry Following cardiac surgery, PCIS occurs in up to one-third of cases.4 postcardiac inju syndrome after cardiac surgery? What is the timing On average, onset of PCIS occurs 3 weeks after surgery but can vary from 3 days to 1 year.4 injury syndrome of postcardiac after cardiac surgery? What are the main The clinical manifestations of PCIS may include fever, chest pain, pericarditis, pleuritis, and pneumonitis.4 clinical manifestations of
postcardiac injury syndrome? How common aren A pleural effusion is present in about 80% of cases of PCIS.4 postcardiac injury pleural effusions i patients with syndrome? What are the s ofs Pleural effusions are bilateral in one-half of patients withlPCIS, and there is a predilection for the left side in cases of unilateral effusion..The fluid is bloody in postcardiac injury pleural effusion associated with characteristic around one-third of patients. PMNs are the dominant cel type in early PCIS, but this evolves to a lymphocytic predominance over time 4 syndrome? What are the key Although CABG can cause PCIS, it can also lead o a separate entity called post-CABG pleural effusion. The time course and fluid parameters are similar for both. clinical differences The main distinguishing features are the absencetof fever and chest pain in patients with post-CABG pleural effusion. Both of these features are present in this case, between consistent with PCIS. 4 postcardiac injury post-CABG pleural syndrome and effusion? What is the r jury There are no known methods of preventing PCIS from developing. Once the diagnosis is made, the cornerstone of treatment includes nonsteroidal anti- postcardiac in syndrome? treatment fo inflammatory drugs (eg, indomethacin 25-50 mg every 8 hours) or glucocorticoids (eg, prednisone). 4
986 Key Points
A pleural effusion isldefined asiexcess fluid in the pleural cavity. The physical findings of pleural effusions include Decreased dullness to percussion, Decreased tactile fremitus, and reduced Symptoms of pleura effusions nclude dyspnea, dry cough, and chest pain (usually pleuritic). inspiratory expansion of the chest wall on the affected side, breath sounds over the effusion. A large pleural effusion occupies >25% of the hemithorax.
A loculated pleural effusion does not flow freely within the pleural An empyema is defined by the presence of pus in the pleural Thoracentesis is helpful in characterizing the nature of pleural Pleural effusions can be transudative or exudative, determined by Basic fluid;features,such as gross appearance; total and differential Supplementary fluid studies (eg, cytologic examination) can be Transudative effusions can occur as a result of changes in cavity and is best evaluated with ultrasonography or CT imaging. space. effusions. Light’s criteria. cell counts protein LDH, and glucose levels; and smears and cultures are helpful in narrowing the differential diagnosis. helpful in certain cases. hydrostatic pressure, changes in oncotic pressure, or diaphragmatic defects.
The most common causes of transudative effusions are congestive heart failure, hepatic hydrothorax, and nephrotic syndrome. Exudative effusions can be infectious or noninfectious.
The most common causes of exudative effusions are pneumonia, pulmonary embolism, and malignancy. Treatment for pleural effusions depends on the underlying
etiology. Some infectious effusions require urgent drainage.
987
References 1. Bhatnagar R, Maskell N. The modern diagnosis and management of pleural effusions. BMJ. 2015;351:h4520.
-
McGrath EE, Anderson PB. Diagnosis of pleural effusion: a systematic approach. Am J Crit Care. 2011;20(2):119-127; quiz 28.
-
Blackmore CC, Black WC, Dallas RV, Crow HC. Pleural fluid volume estimation: a chest radiograph prediction rule. Acad Radiol. 1996;3(2):103-109.
-
Light RW. Pleural effusions following cardiac injury and coronary artery bypass graft surgery. Semin Respir Crit Care Med. 2001;22(6):657-664.
-
Light RW. Parapneumonic effusions and empyema. Proc Am Thorac Soc. 2006;3(1):75- 80.
-
Light RW. Clinical practice. Pleural effusion. N Engl J Med. 2002;346(25):1971-1977.
-
Light RW, Macgregor MI, Luchsinger PC, Ball WC Jr. Pleural effusions: the diagnostic separation of transudates and exudates. Ann Intern Med. 1972;77(4):507-513.
-
Eid AA, Keddissi JI,iSamaha M,lTawk MM, Kimmell K, Kinasewitz GT. Exudative 9. Doustkami H, Hooshyar A, Maleki N, Tavosi Z, Feizi I. Chronic constrictive effusions in congest ve heart fai ure. Chest. 2002;122(5):1518-1523. pericarditis. Case Rep Cardiol. 2013;2013:957497. . Rice TW. Pleural effusions in superior vena cava syndrome: prevalence,
10characteristics, and proposed pathophysiology. Curr Opin Pulm Med. 2007;13(4):324- 327. . Chae EJ, Seo JB, Kim SY, et al. Radiographic and CT findings of thoracic
11complications after pneumonectomy. Radiographics. 2006;26(5):1449-1468. 12. Albores J, Wang T. Images in clinical medicine. Trapped lung. N Engl J Med. 2015;372(19):e25.
-
Jenkins PG, Shelp WD. Recurrent pleural transudate in the nephrotic syndrome. A new approach to treatment. JAMA. 1974;230(4):587-588.
-
Light RW. Pleural Diseases. 5th ed. Philadelphia, PA: Lippincott Williams & Wilkins, A Wolters Kluwer Business; 2007.
-
Cardenas A, Kelleher T, Chopra S. Review article: hepatic hydrothorax. Aliment Pharmacol Ther. 2004;20(3):271-279.
-
Szeto CC, Chow KM. Pathogenesis and management of hydrothorax complicating peritoneal dialysis. Curr Opin Pulm Med. 2004;10(4):315-319.
-
Ferreira PG, Furriel F, Ferreira AJ. Urinothorax as an unusual type of pleural effusion – clinical report and review. Rev Port Pneumol. 2013;19(2):80-83.
-
Sahn SA,:Light RW. The sun should never set on a parapneumonic effusion. Chest. 19. Vorster MJ, Allwood BW, Diacon AH, Koegelenberg CF. Tuberculous pleural 1989;95(5) 945-947. effusions: advances and controversies. J Thorac Dis. 2015;7(6):981-991. . Choi YW Munden RF Erasmus JJ, et al. Effects of radiation therapy on the lung:
20radiologic,appearances,and differential diagnosis. Radiographics. 2004;24(4):985-997; discussion 98.
-
Chapman SJ, Cookson WO, Musk AW, Lee YC. Benign asbestos pleural diseases. Curr Opin Pulm Med. 2003;9(4):266-271.
-
Light RW. Textbook of Pleural Diseases. 3rd ed. Boca Raton, FL: Taylor & Francis Group;
988
2016. 23. Jeon D. Tuberculous pleurisy: an update. Tuberc Respir Dis (Seoul). 2014;76(4):153- 159.
989